Sunteți pe pagina 1din 79

S.S.M.

ROMÂNIA - Filiala Mehedinți 2015

SOCIETATEA DE ȘTIINȚE
MATEMATICE DIN ROMÂNIA
Filiala Mehedinți

REVISTA DE MATEMATICĂ MEHEDINȚEANĂ

R.M.M.

Nr.15-2015

1 REVISTA DE MATEMATICĂ MEHEDINȚEANĂ NR. 15


S.S.M.ROMÂNIA - Filiala Mehedinți 2015

SOCIETATEA DE ȘTIINȚE
MATEMATICE DIN ROMÂNIA
Filiala Mehedinți
COLECTIVUL DE REDACȚIE REVISTA DE MATEMATICĂ MEHEDINȚEANĂ
1 DANIEL SITARU Redactor principal ; Machetare grafică ; Copertă
2 GHEORGHE CĂINICEANU Redactor principal
3 DAN NĂNUȚI Redactor principal
4 EMILIA RĂDUCAN Redactor ediție electronică RMM
5 MIHAI OCTAVIAN UNGUREANU Redactor coordonator primar
6 DANA PAPONIU Redactor coordonator gimnaziu
7 LEONARD GIUGIUC Redactor coordonator liceu
8 IULIANA GIMOIU Redactor clasa a V-a
9 ELENA RÎMNICEANU Redactor clasa a VI-a
10 DRAGA TĂTUCU MARIANA Redactor clasa a VII-a
11 DANIEL STRETCU Redactor clasa a VIII-a
12 CLAUDIA NĂNUȚI Redactor clasa a IX-a
13 DAN NEDEIANU Redactor clasa a X-a
14 GABRIELA BONDOC Redactor clasa a XI-a
15 OVIDIU TICUȘI Redactor clasa a XII-a

2 REVISTA DE MATEMATICĂ MEHEDINȚEANĂ NR. 15


S.S.M.ROMÂNIA - Filiala Mehedinți 2015

CUPRINS
Cuvânt înainte-Prof. Univ. Dr. Radu Gologan………………………………………………………….4
Foști elevi-Gonciulea Constantin-Prof.Dr.Căiniceanu Gheorghe.....................................5

IN MEMORIAM-Profesorul Alexandru Szoros………………………………………………………….5


Metoda reducerii la absurd-Profesor Ecaterina Pupăză...............................................................7

Regula produsului ca metodă de numărare-Profesor Iuliana Gimoiu......................................8

Criterii de divizibilitate-Profesor Sanda Mariana Gorun...........................................................10

O problemă de geometrie-Profesor Marica Ștefan........................................................................16

Geometria triunghiului cu substituțiile Ravi-Profesori Dan Nănuți, Daniel


Sitaru...............................................................................................................................................................20

Geometria complexă a triunghiului echilateral-profesor Claudia Nănuți..........................22


Aplicații ale sistemelor liniare la demonstrarea unor inegalități-profesori Diana
Trăilescu, Leonard Giugiuc....................................................................................................................24

Substituții trigonometrice-profesor Daniel Sitaru........................................................................26

Aplicații ale funcției f(x)=det(Ax+B)-profesor Dan Nedeianu...................................................29

Probleme elementare tratate neelementar-profesor Daniel Sitaru........................................33

Probleme propuse .................................................................................................................... 38


Ciclul primar ............................................................................................................................... 38
Clasa a V-a.................................................................................................................................... 41
Clasa a VI-a .................................................................................................................................. 43
Clasa a VII-a................................................................................................................................. 46
Clasa a VIII-a ............................................................................................................................... 48
Clasa a IX-a .................................................................................................................................. 51
Clasa a X-a.................................................................................................................................... 54
Clasa a XI-a .................................................................................................................................. 57
Clasa a XII-a ................................................................................................................................. 62
Subiecte Concursuri ................................................................................................................. 64
Premianții concursurilor din 2014-2015 ........................................................................... 75

3 REVISTA DE MATEMATICĂ MEHEDINȚEANĂ NR. 15


S.S.M.ROMÂNIA - Filiala Mehedinți 2015

Cuvânt înainte…
Dragi Colegi,

Nu scriu aceste rânduri influenţat de similare discursuri ale politicienilor, ci simplu pentru a vă
mulţumi în primul rând pentru entuziasmul şi dragostea pentru matematică şi educaţia
matematică pe care reuşiţi să o pastraţi, chiar dacă societatea românească nu numai că nu
apreciază dascălul adevărat, dar adesea îl umileşte.

Apartenenţa la Societatea noastră face uneori să mai uităm din problemele ce ne frământa zilnic în
şcoală, prin faptul că, în cei peste o suta de ani de drum împreună, a rămas bucuria de a şti că
gândim la fel şi muncim cu pasiune.

Personal, am un sentiment de mulţumire, sigur nu definitiv, că împreună şi sprijinit de prietenii mei


din filiale şi din Consiliul Director, am reuşit în ultimii ani să reînviem o tradiţie veche şi să mărim
procentul de credinţă în viitorul SSMR. Gazeta a crescut în tiraj, aproape de a face SSMR stabilă,
numărul de membri s-a mărit, iar activităţile şi proiectele aducătoare de sprijin financiar s-au
înmulţit. Mai mult, Societatea Europeană de Matematică ne consideră unul dintre membri cei mai
activi şi importanţi.

S-au revigorat multe filiale, unele cu conduceri extrem de tinere si eficace, s-au revitalizat
legăturile interne cu mediul universitar şi, cel mai important pentru mine, Conferinţele Anuale au
devenit evenimente ştiinţifice cu valoare. Revista noastră de cercetare „Bulletin Mathematique”a
câştigat o importantă cotă în ansamblul ştiinţific mondial.

Anul 2015 ne aduce multe încercări: Olimpadele cu etapele sale, finala de la Bucureşti, Al 7-lea
Congres al Matematicienilor Români de la Iaşi (iunie) unde vom avea o conferinţă satelit de
matematică şcolară şi „problem solving” şi, bineînţeles lupta permanentă de a promova bijuteria
noastră centenară: Gazeta Matematică. De aceea, îi rog pe profesorii implicaţi în distribuţia
Gazetei, acţiune prevăzută în Protocolul SSMR cu Ministerul Educaţiei Naţionale, să continue
această promovare, mai ales că, în acest an, Gazeta aniversează 120 de „toamne”neîntrerupte.
Trebuie să fim mândri că puţine sunt publicaţiile din lume care au o astfel de vârstă fără întreruperi
şi avem datoria faţă de acei „stălpi ai Gazetei”de a continua. Promovaţi apartenenţa la SSMR şi la
GMB.

În încheiere doresc, ca împreună cu colegii mei răspunzători de activitatea în filiale, să vă mai


adresez o rugăminte: ramâneţi în primul rând membri ai societăţii noastre, chiar dacă valurile
apartenenţei la mişcările politice vă fac, uneori, să uitaţi asta. Trebuie sa fim uniţi prin bucuria de a
gândi la fel, corect şi logic, importanţa educaţiei matematice în România.

Cu drag,
Președintele SSMR, prof.univ.dr. Radu Gologan

4 REVISTA DE MATEMATICĂ MEHEDINȚEANĂ NR. 15


S.S.M.ROMÂNIA - Filiala Mehedinți 2015

GONCIULEA CONSTANTIN, CNT 1982-1986

Prezentare de prof.dr.Gheorghe Căiniceanu

Vă prezentăm în acest număr al revistei o biografie a unuia dintre cei mai străluciți elevi ai
Colegiului Național “Traian”, absolvent în anul 1986. Costică era nominalizat adesea “unul
din frații Gonciulea” pentru că și fratele său geamăn Nicolae era un olimpic la matematică
de excepție.Biografia lui Nicolae va apare in paginile unui numar viitor al revistei noastre.
Fideli proiectului inițial de a arăta elevilor noștri exemple de “traiectorii profesionale” vom
începe prin a sublinia că Gonciulea Constantin a fost premiat la o mulțime de concursuri de
matematică: OJM,ONM,”Gh.Titeica”, Concursul rezolvitorilor GMB, Sesiuni Naționale de
comunicări științifice. Una dintre acestea ne-a prilejuit și cunoașterea .Îmi amintesc cu
plăcere că la îndemnul directorului de atunci am efectuat câteva ore de cerc de
matematică la elevii clasei a XII-a și fiind provocat de Constantin cu o problemă din Gazeta
Matematică, am avut discuții în urma cărora a apărut o lucrare premiată la Sesiunea
Națională de la Baia Mare din 1986. Urmează mai jos câteva dintre etapele școlare și
profesionale străbătute de matematicianul-informatician Constantin Gonciulea. Studii: The
Ohio State University,Master's degree, Computer Science,1997 – 1999 ; The Ohio State
University,Doctor of Philosophy (PhD), Mathematics 2000 ; University of Bucharest 1987-
1992 Bachelor's degree, Mathematics and Computer Science ; Distincții și premii in cariera:
2008 Weby Award for Chase Mobile (SMS) application ; 2009 Webby Award for Chase
iPhone application ; 2011 Work featured in Superbowl commercial
2011 Webby Award for Chase Quick Deposit (I did the image processing for it) ; 2011
Expert Engineer at JPMorgan Chase Experiență: Executive Director, Global Technology
,JPMorgan Chase ianuarie 2015 – Prezent (5 luni) ; Technology Director, Global
Technology,JPMorgan Chase ; ianuarie 2012 – decembrie 2014 (3 ani)Columbus, Ohio Area
; Vice President, Global Technology, JPMorgan Chase ; aprilie 2005 – decembrie 2011 (6 ani
9 luni); Webby Awards for SMS, iPhone and Quick Deposit mobile banking applications ;
Consultant,JPMorgan Chase,septembrie 2004 – aprilie 2005 (8 luni) ; Consultant Chemical
Abstracts Services august 2000 – septembrie 2004 (4 ani 2 luni) ; Senior Software
Developer Blue Diesel iunie 1999 – august 2000 (1 an 3 luni)Columbus, Ohio Area;
Publicații:Object Slicing and Component Design with Java(Link),TheServerSide.com,1
septembrie 2004; Virtual Epimorphisms of Coxeter Groups Onto Free Groups(Link) ; Ohio
State University,2000 ; Infinite Coxeter groups virtually surject onto Z(Link),Commentarii
Mathematici Helvetici,1997 ; Atestări:Functional Programming Principles in Scala(Link),Cu
începere din mai 2013 Principles of Reactive Programming(Link),Cu începere din decembrie
2013 Machine Learning(Link),Cu începere din aprilie 2015

5 REVISTA DE MATEMATICĂ MEHEDINȚEANĂ NR. 15


S.S.M.ROMÂNIA - Filiala Mehedinți 2015

IN MEMORIAM
Profesorul Alexandru Szoros (14.01.1946-4.01.2015)
Daniel Sitaru, Președinte SSMR-Filiala Mehedinți
Profesorul Alexandru Szoros s-a născut în comuna Mișca judetul Bihor, unde a
urmat cursurile școlii gimnaziale. A absolvit cursurile liceului din Marghita apoi a urmat
cursurile Facultații de Matematică a Universității Babeș-Bolyai din Cluj. În anul 1988 a
susținut lucrarea metodico-științifică de grad I având titlul “Asupra Ecuației lui Riccati”.
Timp de 30 de ani a activat ca profesor de matematică la Colegiul Tehnic “Lorin Sălăgean”.

Profesorul Alexandru Szoros a fost un om cu un caracter puternic, corect și loial


față de elevii , colegii și meseria sa. De asemenea a fost un membru activ al Societății de
Științe Matematice-Filiala Mehedinți. Creator al multor probleme de o frumusețe
matematică rară, domnul profesor Szoros a fost, de asemenea un propunător prolific de
probleme în principalele reviste de matematică din țară. În cele ce urmează prezentăm
câteva dintre problemele domnului profesor:

1. Rezolvați în mulțimea numerelor naturale ecuația:

+ + ( + )+ ( + )=

2. Rezolvați în ℝ ecuația: +√ − + + −√ − + =

3. Pentru ∈ ℕ, ≥ se notează: = + +⋯+ . Calculați partea întreagă a


( )
numărului: .

4. Dacă ∈ ℕ∗ arătați că numărul +√ + −√ + + este pozitiv și irațional.

5. Aflați valorile parametrului ∈ ℤ pentru care ecuația:

( − − ) + | |+ − − = are soluții întregi.

6. Să se demonstreze că dacă , , ∈ ( , ∞) și ∈ ℕ atunci are loc inegalitatea:


+ + +
+ + ≥
+ + + + + + +
7. Fie , ∈ ℝ∗ . Dacă ecuația ( + ) + ( + ) = + , are o singură soluție
reală, atunci ecuația + ( + ) + ( − ) = are soluții reale distincte.

8. Fiind date numere în intervalul [ , ) arătați că există trei care să fie laturile unui
triunghi.

9. Să se arate că în orice triunghi are loc inegalitatea:



( − )( + )( + )

6 REVISTA DE MATEMATICĂ MEHEDINȚEANĂ NR. 15


S.S.M.ROMÂNIA - Filiala Mehedinți 2015

10. Precizati natura triunghiului în care are loc relația: = .

11. Dacă triunghiul nu este obtuzunghic, arătați că lungimile laturilor sale verifică
inegalitatea:
+ −
+ ≥
+ +
12. Să se arate că în orice triunghi au loc inegalitățile:

+ +
≥ ≥

13. Arătați că în orice triunghi este adevărată dubla inegalitate:

≥ + ≥

14. Să se arate că pentru orice ∈ ℕ∗ avem:

( + ) +
√ + √ + ⋯+ √ ≤ ∙

15. Funcția : ℝ → ℝ are proprietatea că ( ∘ )( ) = + , ∀ ∈ ℝ.


Arătați că este o funcție bijectivă.

16. Să se determine funcția: : ℕ∗ → ( , ∞) cu proprietățile:


( ) ( )
a. ( ) = , b. ∑ ( )= , ∀ ∈ ℕ∗

17. Arătați că: ≤∫ ≤ +


( )
18. Fie : [ , ∞) → ℝ, ( ) = ∫ . Se cere:

a.Rezolvați ecuația: ( ) = . b.Calculați: ∫ ( ) .

( )
19. Fie =∫ si =∫ . Arătați că: { , }≤

7 REVISTA DE MATEMATICĂ MEHEDINȚEANĂ NR. 15


S.S.M.ROMÂNIA - Filiala Mehedinți 2015

Teme pentru grupele de performanță


METODA REDUCERII LA ABSURD

Prof. Ecaterina Pupăză,Colegiul Național “Traian”

Această notă are un caracter metodic și își propune ilustrarea raționamentului prin
reducere la absurd,prin intuiție,încă din primele clase gimnaziale.(chiar primare !)
Repere teoretice:
« Reducerea la absurd » este un procedeu prin care dovedim că o propoziție p este
adevărată, în mod indirect, arătând că negația ei, non p, este falsă.
Practic, pentru a demonstra că propoziția p este adevărată, presupunem că ea ar fi falsă,
apoi analizând consecințele acestei presupuneri, arătăm că se ajunge la un rezultat absurd
sau contradictoriu. Contradicția obținută arată că p nu poate fi falsă.
Invers, dacă avem de demonstrat că propoziția p este falsă, presupunem că ea ar fi
adevărată și arătăm că aceasta presupunere conduce la o contradicție.
Un joc de introducere nonformală a metodei :
Doi copii, sunt provocați la un joc. Ambilor li se arată trei buline ,una roșie și două albastre.
Copiii sunt legați la ochi și fiecaruia i se lipește pe frunte câte o bulină albastră, după care
copiii sunt dezlegați la ochi .Unul dintre ei este întrebat : « Ce culoare are bulina de pe
fruntea ta ? » El răspunde : « Nu știu. »Apoi, i se pune aceeași întrebare celui de-al doilea.
Acesta răspunde: « Albastru ».Cum a judecat?
Soluție :
Dacă pe fruntea mea,colegul ar fi văzut bulina roșie,el ar fi tras concluzia că a lui este
albastră, deci ar fi dat răspunsul. Cum el a spus ca nu știe,bulina mea nu poate fi roșie,deci
este albastră. El a judecat prin reducere la absurd.
Câteva probleme cu bile!
1) Într-o cutie sunt bile albe și roșii. Dacă oricum am extrage cinci bile, vom avea bile de
ambele culori,arătați că nu pot fi mai mult de 8 bile în cutie.
Soluție : Sunt cel mult 4 bile din fiecare culoare.(reducere la absurd).
2) În 10 cutii sunt 84 de bile: roșii, galbene, albastre și verzi. Știind că în fiecare cutie sunt
bile de toate culorile,arătați că există două cutii cu același număr de bile.
Soluție : În fiecare cutie sunt cel puțin 4 bile. Dacă fiecare cutie ar avea un număr diferit de
bile, atunci numărul total ar fi de cel puțin 4 + 5 + 6 + . . . + 13 = 85bile. Contradicție!
3)Avem 13 bile colorate. Arătați că există patru bile colorate la fel sau cinci bile de culori
diferite. Generalizați problema pentru n bile.
Soluție : În caz contrar, ar exista cel mult 4 culori și cel mult 3 bile din fiecare culoare, deci cel
mult 12 bile.
Generalizarea: Fie mn + 1 bile colorate. Atunci, printre ele există m + 1 bile colorate la fel,
sau n + 1 bile de culori diferite.Într-adevar, dacă presupunem că nu există m + 1 bile
colorate la fel, sau n+1 bile de culori diferite, înseamnă că există cel mult n culori și cel mult
m bile din fiecare culoare, deci numărul total de bile este de cel mult mn.
Probleme de concurs
1)Arătați că numerele 1, 2, ..., 16 nu pot fi scrise, o singură dată pe un cerc, astfel încât
suma oricaror două numere alăturate să fie pătrat pefect. Concurs « Gheorghe Țițeica »

8 REVISTA DE MATEMATICĂ MEHEDINȚEANĂ NR. 15


S.S.M.ROMÂNIA - Filiala Mehedinți 2015
Soluție: Presupunem că scrierea ar fi posibilă. Notăm cu x și y numerele vecine lui 16.
Cum x ∈ {1, 2, ..., 15}, vom avea x + 16 ∈ {17, 18, ..., 31}. x + 16 fiind pătrat perfect, rezultă
x + 16 = 25, deci x = 9; analog putem arăta că y = 9.Contradicție!
2) La o masă circulară de joc sunt 8 jucători. La un moment dat, se constată că fiecare
jucator împreună cu cei doi vecini ai săi au un numar impar de fise câștigătoare. Dovediți
că fiecare jucător are cel puțin o fisă câștigătoare. Olimpiada Națională- 2002
Soluție : Presupunem că unul dintre jucători, fie el A, nu are nici o fisă câștigătoare; atunci
unul dintre vecinii lui, notatcu B, are un număr par, iar celalalt,H, un număr impar. Numim
jucatorii, în ordine: A, B, C, D, E, F, G,H. Din enunț¸ avem că lui C îi corespunde un număr
impar, lui D și E câte un număr par, lui F un număr impar și lui G, un număr par. Atunci,
jucatorii vecini F,G ¸si H au împreună un număr par de fise.Contradicție!
3) Justificați dacă există 9 numere naturale prime diferite două câte două,a căror sumă să
fie 125. Olimpiada Locală București-1991.
Soluție: Presupunem că există nouă numere naturale prime diferite două căte două, a căror
sumă să fie 125. Dacă printre cele 9 numere s-ar afla numărul 2,atunci suma ar fi
parăContradicție!Deci suma numerelor este S ≥ 3 + 5 + 7 + 11 + 13 + 17 + 19 + 23 + 29 = 127.
Contradicție! Deci răspunsul problemei este negativ.
Probleme propuse
1) Suma a 10 numere naturale este 62. Arătați că produsul lor se divide cu 60.
2) Demonstrați că pentru orice n ∈N, nenul, cel puțin unul dintre numerele
n, n + 1, n + 2, ..., 2n este pătrat perfect.
3) La un concurs, se știe că în orice grup format din patru participanți, există unul care îi
cunoaște pe ceilalți trei. (Vom considera că dacă X îl cunoaște pe Y, atunci și Y îl cunoaște
pe X). Demonstrați că există un concurent care îi cunoaște pe toți ceilalți.
4) Șase puncte, dintre care oricare trei sunt necoliniare, sunt unite două câte două prin
segmente. Unele segmente se colorează cu roșu, iar celelalte cu albastru. Demonstrați că
există cel puțin un triunghi (cu vârfurile în trei dintre cele șase puncte) cu laturile colorate
la fel.
BIBLIOGRAFIE:
1.A.Engel, «Probleme de matematică. Strategii de rezolvare » Editura Gil-
Zalău-2006.
2.A. Ghioca , L. A. Cojocaru, « Matematica gimnazială dincolo de manual »,
Editura Gil- Zalău- 2005.
3.Colecția Gazeta Matematică – seria B

REGULA PRODUSULUI CA METODĂ DE NUMĂRARE

Prof.Iuliana Gimoiu, Colegiul Național “Traian”


Problemele de numărare sunt considerate probleme dificile atât pentru elevii de liceu, dar
în special pentru cei de gimnaziu. În cele ce urmează vom prezenta fără a depăși un nivel
elementar, câteva aplicații ale regulei produsului în scopul formării și fixări unor abilități
de operare cu aceasta la elevii de gimnaziu.

Regula produsului: Dacă un obiect se poate alege în moduri și dacă după fiecare astfel
de alegere, un obiect se poate alege în moduri, atunci alegerea perechii ( , ) poate fi
9 REVISTA DE MATEMATICĂ MEHEDINȚEANĂ NR. 15
S.S.M.ROMÂNIA - Filiala Mehedinți 2015
realizată în ∙ moduri. Regula produsului poate fi generalizată considerând , , … ,
obiecte ce pot fi alese în , , … , moduri, iar succesiunea alegerii celor obiecte
poate fi efectuată în , ,…, moduri. Ca aplicații directe ale regulei produsului
amintim:

1. Numărul divizorilor unui număr natural este ( + )( + )…( + ), unde


= … este descompunerea lui în factori primi.

Soluție: Orice divizor al lui este de forma = … , unde

∈ {0,1,2, … , }, ∈ {0,1,2, … , }, etc. Putem alege în + 1 moduri, în +1


moduri, etc., iar concluzia rezultă conform regulei produsului.

2. Numărul submulțimilor unei mulțimi cu elemente este .

Soluție: Fie = { , , … , } o mulțime cu elemente. Fiecărei submulțimi a lui îi


vom asocia un −uplu în care pe poziția se va afla 1 dacă ∈ și 0 dacă ∉ . De
exemplu submulțimii { , , } ( )
i se asociază 1,1,0,1,0, … ,0 având componente. Fiecare
submulțime este caracterizată de câte un −uplu în care fiecare componentă poate lua
numai valorile 0 și 1, deci vom obține aplicând regula produsului 2 submulțimi!

3. Un număr se numește „polindrom” dacă el coincide cu răsturnatul său. Câte numere


polindrom de cifre există? Dar de cifre?

Soluție: Numărul polindrom de 5 cifre este bine determinat de primele 3 cifre ale sale, dat
fiind faptul că a patra cifră coincide cu a doua, iar ultima coincide cu prima. Putem alege
prima cifră în 9 moduri, iar a doua și a treia în câte 10 moduri. Astfel obținem 9 ∙ 10 ∙ 10 =
900 numere polindrom de 5 cifre. Se poate justifica asemănător că sunt tot 900 numere
polindrom de 6 cifre.

4. În câte moduri se pot colora căsuțele unui tablou × folosind culori? Dar dacă
dorim ca în plus, tabloul colorat să fie simetric față de diagonala principală?

Soluție: Tabloul având căsuțe, fiecare putând fi colorată în 3 moduri, obținem 3


colorări.

Pentru a doua întrebare observăm că este suficient să colorăm căsuțele diagonalei principale
și pe cele de deasupra ei, celelalte având culoarea deja fixată din condiția de simetrie.
( )
Căsuțele în care avem libertatea de colorare sunt în număr de 1 + 2 + ⋯ + = , deci
( )
obținem 3 colorări.

5. În câte moduri se pot completa cu numerele sau − căsuțele unui tablou × astfel
încât produsul elementelor de pe fiecare linie și de pe fiecare coloană să fie − .

Soluție: Problema are soluție numai dacă atunci când și au aceeași paritate, deoarece
produsul elementelor tabloului poate fi obținut în 2 moduri astfel:

- produsul elementelor pe fiecare linie fiind −1 și cum sunt linii, obținem (−1)

10 REVISTA DE MATEMATICĂ MEHEDINȚEANĂ NR. 15


S.S.M.ROMÂNIA - Filiala Mehedinți 2015
- produsul elementelor pe fiecare coloană fiind −1 și cum sunt coloane, obținem
(−1) , deci (−1) = (−1) , adică și au aceeași paritate.

Pentru a număra în câte moduri putem face completarea,vom considera că fiecare linie
poate fi completată în 2 moduri, deoarece fiecare dintre primele − 1 poziții pot fi
completate în câte 2 moduri, iar ultima poziție ia o singură valoare impusă de condiția ca
produsul elementelor de pe fiecare linie să fie −1. Același lucru se va întâmpla pe fiecare
dintre cele − 1 linii, elementele de pe ultima linie având o singură variantă de completare
impusă de condiția ca produsul elementelor de pe fiecare coloană să fie egală cu −1. Vom
avea atunci 2 ∙2 ∙ …∙ 2 = 2( )( )
modalități de completare a tabloului.

BIBLIOGRAFIE
1. D. Brânzei, G. Popa și colaboratorii – „Pregătirea permanentă a
olimpicilor pentru matematică”, Editura Paralela 45, Pitești, 2005.
2. A.P. Ghioca, Luana Cojocaru – „Matematica gimnazială dincolo de
manual”, Editura Gil, Zalău, 2005
CRITERII DE DIVIZIBILITATE

Prof. Sanda-Mariana Gorun, Liceul teoretic„Traian Lalescu”


Motto: ,,Numerele perfecte sunt foarte rare, ca şi oamenii perfecţi.” Rene Descartes

1. Criteriul de divizibilitate cu 2n şi 5n, n*

Un număr m = a k a k 1...a1a 0 se divide cu 2n respectiv cu 5n, k  n, dacă şi numai dacă


numărul format din ultimele n cifre ale lui m, este divizibil cu 2n respectiv cu 5n.

Demonstraţie: Numărul m se scrie în baza 10 sub forma: m = ak  10k + ak-1  10k-1 + ….+ an 
10n + a n 1a n 2 ...a1a 0 . Deoarece 2n / 10k (5n / 10k) pentru orice k  n, rezultă că 2n / m (5n / m)
dacă şi numai dacă 2n / a n 1a n 2 ...a1a 0 (5n / a n 1a n 2 ...a1a 0 ).

2. Criteriul de divizibilitate cu 7, 11, 13

Un număr natural se divide cu 7 ( sau 11, sau 13) dacă şi numai dacă diferenţa dintre cele
două numere naturale obţinute prin „tăierea” numărului dat în două astfel încât la dreapta
să rămână un număr de 3 cifre, este divizibilă cu 7 (sau 11, sau 13).

Demonstraţie: Fie m = a n a n 1...a 2a1a 0 , n , n  2 şi p = a n a n 1...a 3 , q = a 2a1a 0 .

Atunci m = 103  p + q = (71113 – 1)p + q = 71113p + q – p. Rezultă că 7 / m dacă şi numai


dacă 7/ (q – p).

11 REVISTA DE MATEMATICĂ MEHEDINȚEANĂ NR. 15


S.S.M.ROMÂNIA - Filiala Mehedinți 2015

Exemplu: Să arătăm că numărul 83564 se divide cu 13; 564 – 83 = 481, deci 481  13.

3. Criteriul de divizibilitate cu 11

Un număr natural se divide cu 11 dacă şi numai dacă diferenţa dintre suma cifrelor de rang
par şi suma cifrelor de rang impar din numărul dat, se divide cu 11.

Demonstraţie: Fie m = a n a n 1...a 2a1a 0 = a n  10n + a n-1  10n-1 + ….+ a1  10 + a0 şi p = (a0 + a2 +


... ) – (a1 + a3 …). Dacă r = 2k, atunci 10r = 102k = 9 111 … 1 + 1 = 9  M11 + 1.

Dacă r = 2k + 1, atunci 10r = 102k+1 = 1000 … 01– 1 =9090 … 9091  11 – 1 = M11 – 1.

Rezultă că m = p + M11 şi deci 11 / m dacă şi numai dacă 11 / p.

Exemplu: Fie numărul 72424. p = 4 + 4 + 7 – (2 + 2) = 112.

3’. Divizibilitatea cu 13

Despărţim numărul dat în grupe de către trei cifre de la dreapta la stânga şi înmulţim cifrele
grupelor de rang impar cu 1,-3 respectiv -4, iar pe cele ale grupelor de rang par cu -1,3
respectiv 4.Dacă suma algebrică a acestor produse se divide cu 13 şi numărul dat se divide cu
13.

4. Criteriul de divizibilitate cu 3, 7 şi 19

Un număr natural se divide cu 3 (sau 7, sau 19) dacă şi numai dacă suma dintre numărul
format din ultimele două cifre mărit de 4 ori şi numărul format din celelalte cifre, este
divizibilă cu 3 (sau 7, sau 19).

Obs. Dacă este necesar se repetă procedeul până când se obţine un rezultat a cărui
divizibilitate cu 3 sau 7 sau 19 este evidentă.

Demonstraţie: Fie m = a n a n 1...a 2a1a 0 , n N, n  2 şi p = a n a n 1...a 2 , q = a1a 0 .

Atunci 4m = 4102  p + 4q = (3719 + 1)p + 4q = 3719p + p + 4q. Rezultă că 19 / m dacă şi


numai dacă 19/ (p + 4q).

Exemplu: Fie numărul 1110987 ; 11109 + 4  87 = 11457; 114 + 4  57 = 342; 3 + 4  42 = 171


iar 171  19.

5. Criteriul de divizibilitate cu 19

Un număr natural se divide cu 19 dacă şi numai dacă suma dintre dublul cifrei unităţilor şi
numărul format din celelalte cifre, este divizibilă cu 19.

12 REVISTA DE MATEMATICĂ MEHEDINȚEANĂ NR. 15


S.S.M.ROMÂNIA - Filiala Mehedinți 2015

Demonstraţie: Fie m = a n a n 1...a 2a1a 0 , n , n  1 şi p = a n a n 1...a1 .

21m = 210p + 21a0 = (11  19 + 1)p + (19 + 2) a0 = M19 + p + 2a0. Cum (21, 19) = 1, avem că 19 /
m dacă şi numai dacă 19 / p + 2a0.
Exemplu: Fie numărul 1110987. 111098 + 2  7 = 111112 ; 11111 + 2  2 = 11115 ; 1111 + 2 
5 = 1121; 112 + 2  1 = 114; 11 + 2  4 = 19 iar 19  19

6. Criteriul de divizibilitate cu 27 şi 37

Un număr natural se divide cu 27, respectiv 37 dacă şi numai dacă suma numerelor
naturale obţinute prin „tăierea” numărului în grupe de câte 3 cifre, începând de la dreapta,
se divide cu 27, respectiv cu 37.

Demonstraţie: Fie m = a n a n 1...a 2a1a 0 . Atunci m = a 2a1a 0 + a 5a 4a 3  103 + …..+ a n a n 1a n 2 


10n-2. Cum 103 = 27  37 + 1, avem m = M37 + a 2a1a 0 + a 5a 4a 3 + …+ a n a n 1a n 2 .

Deci 37 / m dacă şi numai dacă 37 / a 2a1a 0 + a 5a 4 a 3 + …+ a n a n 1a n 2 .

Exemplu: Fie numărul 5392158 ; 158 + 392 + 5 = 555 iar 37 / 555

7. Criteriu general de divizibilitate

Un număr natural m = a n a n 1...a 2a1a 0 se divide cu 10p  q, n, p, q  *, dacă şi numai dacă
înlăturând ultima cifră, înmulţind numărul obţinut cu q şi scăzând (adunând) la noul număr
de p ori cifra suprimată, se obţine un număr divizibil cu 10p  q.

Demonstraţie: Efectuând operaţiile indicate se obţine numărul m1 = (10n-1 an + 10n-2  an-1 +


… + a1)  q  p a0 . Atunci 10m1 - qm =  (10p  q) a0. Rezultă că 10p  q / m dacă şi numai
dacă 10p  q / m1.

Exemplu: Să se verifice dacă numărul 232716 se divide cu 43.

43 = 10  4 + 3, deci p = 4 şi q = 3 ; m1 = 3  23271 – 4  6 = 69789 ; m2 = 3  6978 – 4  9 =


20898, m3 = 3  2089 – 4  8 = 6235 ; m4 = 3  623 – 4  5 = 1849 ; m5 = 3  184 – 4  9 = 516 ; m6
= 3  51 – 4  6 = 129; 129  43.

APLICAŢII

1. Arătaţi că numărul este divizibil prin 8 dacă şi numai dacă 4a + 2b + c este divizibil
cu 8. Generalizare: un număr natural este divizibil cu 8 dacă şi numai dacă suma dintre
cifra unităţilor, dublul cifrei zecilor şi cifra sutelor mărită de 4 ori, este divizibilă cu 8.

13 REVISTA DE MATEMATICĂ MEHEDINȚEANĂ NR. 15


S.S.M.ROMÂNIA - Filiala Mehedinți 2015

Soluţie: abc = 100a + 10b + c = 8(12a + b) + (4a + 2b + c). Deci 8/ abc dacă şi numai dacă 8/
4a + 2b + c. a n a n 1...a 2a1a 0 = an  10n + an-1  10n-1 + ….+ 100 a2 + 10 a1 + a0 = an  10n + an-1 
10n-1 + ….+ a3  103 + 96 a2 + 8 a1 + (a0 + 2a1 + 4a2). Deoarece 8 / a n  10n + an-1  10n-1 + ….+ a3 
103 + 96 a2 + 8 a1, rezultă că 8 / a n a n 1...a 2a1a 0 dacă şi numai dacă 8 / a0 + 2a1 + 4a2.

2. Arătaţi că dacă şi numai dacă .

Soluţie: Fie A = abcxzy = a  105 + b  104 + c 103 + x 102 + y 10 + z

şi B = bcxzya = b  105 + c 104 + x 103 + y 102 + z 10 + a.

Observăm că B = 10A – 999999a. Folosind criteriul de divizibilitate cu 37 obţinem că

999999  37, iar (37, 10) = 1, deci 37 / A dacă şi numai dacă 37 / B.

3. Arătaţi că numărul se divide cu 7 dacă suma cifrelor numărului este 7.

Soluţie: Numărul abb  7 dacă şi numai dacă 4 bb + a  7. Cum 4 bb + a = 44b + a = 44b + 7 –


2b = 7(6b + 1)  7.

4. Să se determine numerele naturale formate din patru cifre impare diferite, care sunt
divizibile cu 21.

Soluţie: Fie n = abcd , a, b, c, d 1, 3, 5, 7, 9, a  b  c  d.

Cum 3 / n implică 3 / (a + b + c + d)  (a, b, c, d)  (1, 3, 5, 9), (3, 5, 7, 9), …... Avem astfel
4! + 4! = 48 de numere n cu proprietatea 3 / n. Dintre acestea, folosind criteriul de
divizibilitate cu 7, găsim pe cele divizibile cu 7. De exemplu 5397 (397 – 5 = 392  7).

5. Să se arate că numărul = … … … , a  0, k, p , nu poate fi prim.

Soluţie: Numărul n are 2(k + p + 1) cifre, iar diferenţa dintre suma cifrelor de rang par şi
suma cifrelor de rang impar este 0, deci 11 / n şi n  11.

6. Fie numărul = … de n + 2 cifre şi numărul = … de 3n


cifre, n*. Arătaţi că, dacă 2 / n, atunci 11 / A şi 11 / B.

Soluţie: Dacă n = 2k avem A = a(102k+1 + 1) + b111 … 10 . Cum 11 / (102k+1 + 1) şi

11 111 … 10 rezultă că 11|A. Avem B = abb ( 106k – 3 + 106k – 6 + ….+ 103+ 1). Cum 11|( 106k –
3
+ 106k – 6 + ….+ 103 + 1) rezultă că 11|B.

14 REVISTA DE MATEMATICĂ MEHEDINȚEANĂ NR. 15


S.S.M.ROMÂNIA - Filiala Mehedinți 2015
7. Fie numărul n = 1234567891011.…..9899. Stabiliţi dacă n se divide cu 11.

Soluţie: Numărul n are 91 + 902 = 189 cifre. Suma cifrelor de rang impar este S1 = 1 + 3 + 5 +
7 + 9 + 9(0 + 1 + 2 +….+ 9) = 430 iar suma cifrelor de rang par este S2 = 2 + 4 + 6 + 8 + 101 +
10  2 + …. + 10 9 = 470. Cum S2 – S1 = 40, numărul n nu se divide cu 11.

Cele mai simple criterii de divizibilitate cu 7 sau 7 criterii de divizibilitate cu 7

Un număr este divizibil cu 7, 11, 13 dacă diferența dintre numărul format din ultimele 3
cifre ale numărului dat și cel rămas este divizibilă cu 7, 11, 13.

CRITERIUL 1: Se scrie numărul in baza 10 folosind puterile lui 10, se înlocuieşte numărul 10
cu 3 şi se fac calculele; Dacă rezultatul obţinut se divide cu 7,atunci şi numărul iniţial se
divide cu 7. Exemplu: fie numărul 5285; in baza 10 se scrie: 5.10 3 +2.10 2 +8.10+5 şi prin
înlocuirea bazei 10 cu 3 se obţine 5.3 3 +2.3 2 +8.3+5 = 182  7.deci 5285  7.

CRITERIUL 2: ( o variantă a primei reguli): Se înmulţeşte prima cifră din stânga cu 3 şi se


adună cu cifra următoare;rezultatul se înmulţeşte cu 3 şi se adună cifra următoare ş.a.m.d.
până la ultima cifră. Pentru simplificarea rezultatului se admite ca după fiecare operaţie să
se scadă ,din rezultatul obţinut 7 sau multiplu de 7. Exemplu: Fie numărul 5285; operaţiile
sunt următoarele: 5∙3 =15 , 15+2=17,dar 17=7∙2+3;se renunţă la 7∙2 şi se continuă
3∙3+8=17,17=7∙2+3; 3∙3+5=14  7

CRITERIUL 3: Vom proceda ca la regula precedentă dar vom începe înmulţirea de la cifra
unităţilor cu 5 de această dată:să exemplificăm pentru numărul 48902,2∙5=10=7∙1+3;
(3+0)∙5=15=7∙2+1; (1+9)∙5 =50 =7∙7+1; ( 1+8)∙5=45 =7∙6+3, 3+4 =7,deci numărul 48902  7

CRITERIUL 4: Se dublează cifra unităţilor şi se scade din rezultat cifra zecilor; din nou se
dublează rezultatul apoi se adună cu cifra sutelor;procedeul se continuă alternând
scăderea cu adunarea.Acolo unde este posibil rezultatul se poate micșora cu un multiplu al
lui 7. Exemplu:Fie numărul 5943,3∙2= 6 , 6- 4=2, 2∙2 =4, 4+9 =13, 13= 7+6 ,6∙2 =12, 12-5
=7,deci numărul 5943  7 .

CRITERIUL 5: Este o regulă comună a divizibilităţii cu 7, 11, 13. Se împarte numărul in clase:
clasa unităţilor, clasa miilor, clasa milioanelor,etc. Dacă diferenţa sumelor grupelor
numărului dat ,adunate din 2 în 2, se divide cu 7,cu 11 sau cu13, atunci numărul se divide
cu 7, 11 sau 13. Exemplu: aplicăm regula pentru numărul 55285783 , (783+55) –285 =553
este divizibil cu 7.

CRITERIUL 6: Este o regulă comună a divizibilităţii cu 7, cu 3 sau cu 19.Se dau deoparte


ultimele două cifre ale numărului , iar la numărul rămas se adună numărul format din cele
două cifre date deoparte înmultit cu 4; dacă e necesar se repetă procedeulpânăse obţine
un rezultat a cărui divizibilitate cu 3, cu 7 cu 19 este evidentă.

15 REVISTA DE MATEMATICĂ MEHEDINȚEANĂ NR. 15


S.S.M.ROMÂNIA - Filiala Mehedinți 2015
Exemplu: Fie numărul 134064,64∙4 = 256, 1340+256 = 1596; repetăm regula : 96∙4 =384 și
15+384 = 399, numărul 399 se divide cu 7 şi cu 3.

CRITERIUL 7 :Numărul natural N se divide cu 7 ( cu 11 si cu 13) dacă și numai dacă diferența


nenegativă dintre cele două numere obținute din numărul natural dat prin tăierea lui în
două, astfel ca la dreapta să ramână trei cifre, se divide cu 7 ( cu 11 sau 13). Dacă numărul
are mai mullt de 6 cifre,împărțim de la dreapta la stânga numărul în grupe de câte trei cifre
Dacă diferenţa dintre suma numerelor exprimate prin grupe de rang par şi suma grupelor
de rang impar se divide cu7, 11, 13, numărul dat se divide cu 7, 11, 13. Sau: a.Dacă un
număr de două cifre se divide cu 7, atunci numărul format din aceleaşi cifre scrise in
ordine inversă, mărit cu cifra zecilor din numărul iniţial se divide cu 7. Exemplu: 63  7;
prin urmare numărul 36+6 = 42  7. b.Dacă un număr de trei cifre se divide cu 7 ,atunci
numărul format din aceleaşi cifre scrise in ordine inversă, micşorat cu diferenta dintre cifra
unităţilor si a sutelor numărului iniţial,se divide cu 7. Exemplu: numărul 126 se divide cu
7.Numărul 621—(6-1)=616 se divide cu 7. c.Dacă suma cifrelor unui număr cu trei cifre este
egală cu 7,el se divide cu 7 numai dacă cifra zecilor este egală cu cifra unităţilor. Exemplu:
322 se divide cu 7 deoarece 3+2+2 = 7.

Probleme propuse

1) Numerele naturale de la 1 la 2010 sunt aranjate ca mai jos. Pe ce rând şi coloană este
numărul 2010?
Rândul 1: 3 11 19

Rândul 2 : 2 6 10 14 18 22 

Rândul 3 :1 5 9 13 17 21 25 …

Rândul 4 : 4 8 12 16 20 24 

Rândul 5 : 7 15 23 OL Neamț
2010

2) a) Arătaţi că numărul = ∙ + ∙ − este divizibil cu 1296,


unde n este un număr natural nenul.

b) Găsiţi cel mai mic număr natural a cărui ultimă cifră este 6, astfel încât dacă ultima

cifră este mutată în faţa numărului (exemplu: 14576 prin mutare devine 61457)

acesta se măreşte de 4 ori.

3) Determinaţi cifra a în baza zece astfel încât să existe relaţia

, ( )+ , ( )= , ( )+ , ( ); OL Vaslui 2010

16 REVISTA DE MATEMATICĂ MEHEDINȚEANĂ NR. 15


S.S.M.ROMÂNIA - Filiala Mehedinți 2015
4) Suma tuturor numerelor naturale de trei cifre diferite între ele, cu cifrele a,b,c, este
divizibilă cu a+b+c? OLM Suceava, 1985

5) Stabiliţi dacă numărul = + + este divizibil cu 7. OLM Galaţi, 1993

6) Aflaţi câtul împărţirii numărului 2002 prin numărul natural a, dacă restul este egal cu

4-a. Prof. Sorin Peligrad

7) Prin împărţirea numerelor , , la acelaşi număr natural, obţinem câturile


, , şi resturile a,b,c. Aflaţi împărţitorul. OJM Hundedoara, 2000

O PROBLEMĂ DE GEOMETRIE

Prof. Marica Ștefan


Se dă trapezul , cu ∥ și = °. Dacă ⊥ ; = ; =
determinați lungimea laturii .

Soluția 1: (bazată pe asemănare)

Notăm = 1; = 2; 1 = ; 2 =
3
∆ ~∆ ⇒ = = ⇒ = = ⇒4 =
12
În ∆ : = ∙ ⇒ 12 = ( + )
√ √
144 = (4 + ) ∙ 4 ⇒ = ⇒ = ; = + = 6√5

În ∆ : = − = 6√5 − 12 = 36 ⇒ =6

Soluția 2: (bazată pe coliniaritate)

17 REVISTA DE MATEMATICĂ MEHEDINȚEANĂ NR. 15


S.S.M.ROMÂNIA - Filiala Mehedinți 2015

În ∆ ; − mediană; = = 6 ; În ∆ ; − mediană; = = 1,5

≡ ≡ ≡ ⇒ coliniare și = + = 6 + 1,5 = 7,5. Deducem


∥ . În ∆ : = − = 7,5 − (6 − 1,5) = 36 ⇒ =6

Soluția 3: (bazată pe trigonometrie)

= ; = ; tg = ; tg = ;

tg ∙ tg = ; tg ∙ ctg = ⇒ ∙ = ; =6
∙ ∙
Soluția 4: (analitică)

: = ; : = ; ( − 4 ) = −4 ; =− +

Din ⊥ ⇒ =− ⇒ ∙ = −1; =4 ⇒ =2 .

( )∙ ( )∙
(0,2 ) ⇒ =2 =2∙3=6; [ ]= = =5

Problema 1: Se dă trapezul cu ∥ și = °.

Știind că = ; = și = să se afle ; .

Soluție:

18 REVISTA DE MATEMATICĂ MEHEDINȚEANĂ NR. 15


S.S.M.ROMÂNIA - Filiala Mehedinți 2015
( + )∙ (4 + ) ∙ 2
[ ]= = =5
2 2
∙ ∙ sin 2 √5 ∙ √5 ∙ sin
[ ]= =
2 2

= 16 +4 = 2 √5; = 4 + = √5

Din 5 sin =5 ⇒ sin =1⇒ = 90°

Problema 2: Se dă trapezul cu ∥ și = °

√ √
Dacă = ; = ; = ; = ; = ∩ ; = să se
[ ]
afle raportul [ ]
.

Soluție:

[ ] √ √
∆ ~∆ ⇒ [ ]
= ; = ; = ; − = − 6√5 =

9√5 + 15 9√5 − 15 15 + 3√5 15 − 3√5


= ∙ ∙ ∙ = 45
2 2 2 2
= 45
( ) ⇒ ℎ = 45 ⇒ ℎ = = 6; = − = 6√5 − (2 ∙ 6) = 144
=

= 36 ∙ 5 − 144 = 36 ⇒ = 6 ; Rezultă − trapez dreptunghic

În ∆ : = + = 36 + 81 = 9 ∙ 13 ; = 3√13; [ ] = 3 7 + √13

Problema 3: Să se afle aria trapezului în care: + = ; = √ ;

= √ ; > 0.

Soluție:

19 REVISTA DE MATEMATICĂ MEHEDINȚEANĂ NR. 15


S.S.M.ROMÂNIA - Filiala Mehedinți 2015
+ + +
= [ ]; =

√ √ √ √
= ; − = ; − = ; − − = ;

= [( − )( − )] = ∙ ; = ⇒ =

Problema 4: Fie un trapez ortodiagonal în care ∥ ; = °;

= ; = . Să se afle aria semidiscului înscris în ∆ și cu diametrul pe .

Soluție: Fie bisectoarea ∢ ; ∈( ). Ducem ⊥ și ⊥ .

∆ ~∆ ⇒ = 1 1 + 1 1
⇒ + = ⇒ + =1
∆ ~∆ ⇒ =
1 1 4 4 8
⇒ + =1⇒ = ; = = =
4 2 3 2 3 9

GEOMETRIA TRIUNGHIULUI CU SUBSTITUȚIILE RAVI

Prof. Dan Nănuți, Colegiul Național „Traian”

Prof.Daniel Sitaru, Colegiul Național Economic „Theodor Costescu”


Fie , , numere reale strict pozitive. Notăm = + ; = + ; = + și în acest
fel , , sunt laturile unui triunghi. Prin adunarea relațiilor obținem:

+ + = ( + + ) și apoi = ( + + ); = + + . Se deduce apoi că:


= − ; = − ; = − , de unde: = − ; = − ; = − .

Aria a triunghiului devine: = ( − )( − )( − ) = ( + + )

( )
Raza cercului înscris în triunghi este: = = =

20 REVISTA DE MATEMATICĂ MEHEDINȚEANĂ NR. 15


S.S.M.ROMÂNIA - Filiala Mehedinți 2015
( )( )( )
Raza cercului circumscris triunghiului este: = =

Formulele fundamentale pentru unghiuri:

( )( ) ( )( )
= = ( )( )
; = = ( )( )

( )( ) ( ) ( )
= = ( )( )
; = = ( )( )

( ) ( ) ( ) ( )
= = ( )( )
; = = ( )( )

( − )( − )
= = =
( − ) ( + + ) + +

( − )( − )
= =
( − ) ( + + )

( − )( − )
= = =
( − ) ( + + ) + +

Lungimile bisectoarelor și înălțimilor triunghiului sunt date de:

= ( − )= ( + )( + )( + + )
+ + +

= ( − )= ( + )( + )( + + )
+ + +

= ( − )= ( + )( + )( + + )
+ + +
( ) ( ) ( )
= = ; = = ; = =

BIBLIOGRAFIE:
1. Leonard Giugiuc – „The Dual Ravi’s Theorem”-RMT-2/2015
2. Daniel Sitaru – „Probleme de geometrie afină și euclidiană”- Editura Ecko
Print-2013 – Drobeta Turnu Severin

21 REVISTA DE MATEMATICĂ MEHEDINȚEANĂ NR. 15


S.S.M.ROMÂNIA - Filiala Mehedinți 2015

GEOMETRIA COMPLEXĂ A TRIUNGHIULUI ECHILATERAL

Prof.Claudia Nănuți, Colegiul Național Economic „Theodor Costescu”


Fie , , ∈ ℂ, distincte. Numerele , , sunt afixele vârfurilor unui triunghi
echilateral dacă și numai dacă:

1. + + = ; ≠ ; =
2. − + ( − ) + ( − ) = ; ≠ ; =
3. + + ; + + ; + + ; ≠ ; sunt afixele vârfurilor unui
triunghi echilateral
4. ± ; ± ; ± sunt afixele vârfurilor unui triunghi echilateral
5. + + = + +
6. ( − ) +( − ) +( − ) =
7. + + =

8. = și ≠ sau ≠ sau ≠

9. | | = | | = | | = > 0; + + =
10. ( − )( − ) = ( − )
( − )( − ) = ( − )

Demonstrația 1: Fie afixul centrului cercului circumscris.

= +( − ) ; = +( − ) = +( − ) unde

+ + = ; =

+ + = + [ +( − ) ]+ [ +( − ) ]=

= + ( + )+( − ) +( − ) = − + ( − )+ ( − )=

=( − )( + + )=

Reciproc: + + = ; + + =

⇒ ( − )+ ( − )= ⇒| − |=| − |

+ + = ; + + = ⇒ ( − )+ − = ⇒| − |
=| − |

, , distincte ⇒ | − |=| − |=| − |≠

Demonstrația 2: − + ( − )+ ( − )= se scrie

+ + −( + + )=

+ + − = , + + − =

22 REVISTA DE MATEMATICĂ MEHEDINȚEANĂ NR. 15


S.S.M.ROMÂNIA - Filiala Mehedinți 2015

( + + ) − = ⇒ + + =

și din demonstrația 1 rezultă cerința.

Demonstrația 3: ( + + )+ ( + + )+ ( + + )=

+ + + + + + ( + + )=

( + + )( + + )=

+ + = și din demonstrația 1 rezultă cerința.

Demonstrația 4:

Din demonstrația 1 rezultă + + = și prin înmulțire cu rezultă + +


= . Prin adunarea respectiv scăderea lor rezultă:

± + ( ± )± ( ± )= și din demonstrația 1 rezultă cerința.

Reciproc, ± , ± , ± fiind afixele vârfurilor unui triunghi echilateral.

± + ( ± )+ ( ± )= , + + ±( + + )=

+ + ± ( + + )= ,( + + )( ± )=

+ + = și din demonstrația 1 rezultă cerința

Demonstrația 5: + + = + + ,

= + + − − − , =( + + )( + + )

de unde + + = sau + + = și din demonstrația 1 rezultă cerința.

Demonstrația 6: ( − ) −( − ) +( − ) =

+ + − − − =

+ + = + + și din demonstrația 5 rezultă cerința

Demonstrația 7:

+ + =
− − −
( − )( − )+( − )( − )+( − )( − )=

− − + + − − + + − − ++ =

+ + = + +

și din demonstrația 5 rezultă cerința.

23 REVISTA DE MATEMATICĂ MEHEDINȚEANĂ NR. 15


S.S.M.ROMÂNIA - Filiala Mehedinți 2015
Demonstrația 8:

= ⇒ − − =
− −

⇒( − )( − )=( − )( − ), + + = + +

și din demonstrația 5 rezultă cerința.

Demonstrația 9: ∙ = ; ∙ = ; ∙ =

+ + = ⇒ + + = ⇒

⇒ + + = ⇒ + + =

⇒ + + =( + + ) − ( + + )=

⇒ + + = + + și din demonstrația 5 rezultă cerința.

Demonstrația 10: Prin trecere la module obținem:

| − |∙| − |=| − |

| − |∙| − |=| − | de unde rezultă | − |=| − |=| − |>0

Reciproc: dacă = + atunci − =( − ) +

− =( − ) − și rezultă: ( − )( − ) = −( − )

( − )( − ) = −( − )

BIBLIOGRAFIE:
1. Colecția Gazeta Matematică – seria B
2. Daniel Sitaru – „Geometrie hipercomplexă și cuaternionică” – Editura
Ecko Print - 2013
APLICAȚII ALE SISTEMELOR LINIARE LA DEMONSTRAREA
UNOR INEGALITĂȚI

Prof. Diana Trăilescu, Prof. Leonard Giugiuc-Colegiul Național “Traian”


Domeniul (destul de larg, de altfel) al inegalităților se rezumă în general fie la inegalități
simetrice (și eventual omogene), fie la inegalități circulare (cunoscute și ca inegalități
ciclice). Numărul unu mondial în domeniu, românul Vasile Cirtoaje, a deschis drumul către

24 REVISTA DE MATEMATICĂ MEHEDINȚEANĂ NR. 15


S.S.M.ROMÂNIA - Filiala Mehedinți 2015
un anumit gen de inegalități atipice. În cele ce urmează, vom demonstra două astfel de
inegalități printr-o metodă nouă, descoperită de autorii acestui articol.

Problema 1 (Vasile Cîrtoaje): Demonstrați că pentru orice numere reale , , cu


≠ ≠ ≠ are loc inegalitatea:

+ + ≥ .

Problema 2 (Beijing Buletine Matematique): Demonstrați că pentru orice numere


reale , , cu ≠ ≠ ≠ are loc inegalitatea:

+ + + + + ≥ .

Evident că o abordare directă, prin calcul algebric elementar, nu duce nicăieri, ci


dimpotrivă, face rezolvarea imposibilă. Astfel a apărut idea unor substituții, acestea
ducând la studiul unor sisteme liniare șiomogene. Vom reaminti, fără demonstrație,
următorul rezultat clasic:

Teoremă: Fie matricea ∈ ( ) și sistemul omogen ∶ ∙ = . Atunci admite

soluții netriviale = dacă și numai dacă = .

Soluția problemei 1 (Diana Trailescu): Notăm = , = și = . Obținem



sistemul liniar și omogen ∙ = , unde = − . Cum ≠ ≠ ≠

, sistemul admite în mod necesar soluții nebanale, deci

= ⟹ + + =− . + + − = + + +
( + + )=( + + ) ≥ .

Remarcă: Se poate observa la cazurile de egalitate o infinitudine destul de arbitrară de


situații.

Soluția problemei 2 (Leonard Giugiuc): Notăm = , = și = . Obținem




sistemul liniar și omogen ∙ = , unde = − . La fel ca mai

sus, este necesar ca sistemul să admită soluții nenule, deci + + =( + + )−
. Notăm + + = . Obținem:

+ + + + + −
− − −
= ( + ) +( + ) +( + ) − =

25 REVISTA DE MATEMATICĂ MEHEDINȚEANĂ NR. 15


S.S.M.ROMÂNIA - Filiala Mehedinți 2015

= + + + + = − ( + + )+ + = − ( − )+ + =

= + + = ( + ) ≥ .

La fel ca la prima problemă, observăm o situație destul de bizară de cazuri de


egalitate.Vom propune spre rezolvare următoarele aplicații: Demonstrați că pentru orice
numere reale , , cu ≠ ≠ ≠ au loc inegalitățile:

( ) + + ≥ ;
− − −

( ) + + + + + ≥ + + ;
− − −
+ + +
( ) + + ≥ .
− − −

( )(Michael Rozenberg, Israel) + + ≥

În încheiere, vom prezenta o bibliografie atipică, adică două dintre cele mai mari forumuri
interactive de matematică din lume, ultimul fiind administrat de autorii acestui
articol.Acestea sunt:
( ) ∶ http://www.artofproblemsolving.com/community/c6t322f6_inequalities_proposed

( ) ∶ https://www.facebook.com/groups/ShMaId/

SUBSTITUȚII TRIGONOMETRICE

Prof.Daniel Sitaru, Colegiul Național Economic „Theodor Costescu”


În cele ce urmează vom utiliza substituții trigonometrice pentru rezolvarea unor ecuații și
pentru calculul termenului general al unor șiruri.

1. Să se rezolve ecuația:
− + ( − ) = ; ∈ℝ

Soluție: ∈ ℝ ⟹ ∃ ∈ − , ; =

3 −
(1 − 3 )=3 − ⟹ =
1−3
1
= 3 ⟹3 = + ; ∈ℤ⟹ = + ; ∈ℤ
3 3
1
= = + ; ∈ {−2,0,2}
3 3

26 REVISTA DE MATEMATICĂ MEHEDINȚEANĂ NR. 15


S.S.M.ROMÂNIA - Filiala Mehedinți 2015
Dacă = 1 ecuația: −3 − 3 + 1 = 0 are soluțiile:

= + ; ∈ {−2,0,2} ; = ; = ; = −

Dacă = 2 ecuația: −6 − 3 + 2 = 0 are soluțiile: = 2+ ;

∈ {−2,0,2} ; = 2 ; = 2+ ; = 2−

2. Să se rezolve ecuația:

− + ( − + ) = ; ∈ℝ

Soluție: ∈ℝ⟹∃ ∈ − , ; = ; ( −6 + 1) = 4 −4

= ⟹ = 4 ⟹ = ; = + ; ∈ℤ

Dacă = 1 ecuația: +4 −6 − 4 + 1 = 0 are soluțiile:


5 3 9
= ; = ; = − ; =
16 16 16 16
3. Să se determine formula termenului general al șirului dat de relațiile:

= ; ∈ [− , ]; = − ; ≥
Soluție: = ∈ [−1,1] ⇒ (∃) ∈ [0,2 ]; = cos ; = arccos

=2 − 1 = 2cos − 1 = cos(2 ) ; =2 − 1 = 2cos (2 ) − 1 = cos(2 )

=2 − 1 = 2cos (2 ) − 1 = cos(2 ). Prin inducție se demonstrează că:

= cos(2 ) = cos(2 arccos )

4. Să se determine formula termenului general al șirului dat de relațiile:

= ; ∈ [− , ]; = − ; ≥
Soluție: = ∈ [−1,1] ⇒ (∃ ) ∈ [0,2 ]; = sin ; = arcsin

=1−2 = 1 − 2cos = cos(2 ) ; =1−2 = 1 − 2cos (2 ) = cos(2 )

=1−2 = 1 − 2cos (2 ) = cos(2 )

Prin inducție se demonstrează că: = cos(2 ) = cos(2 arcsin )

5. Să se determine formula termenului general al șirului dat de relațiile:

= ; ∈ [− , ]; = ( − ) ; ≥

Soluție: = ∈ [−1,1] ⇒ (∃) ∈ [0,2 ]; = sin ; = arcsin

27 REVISTA DE MATEMATICĂ MEHEDINȚEANĂ NR. 15


S.S.M.ROMÂNIA - Filiala Mehedinți 2015
= (3 − 4 ) = sin (3 − 4sin ) = sin(3 )

= (3 − 4 ) = sin 3 (3 − 4sin 3 ) = sin(3 )

= (3 − 4 ) = sin(3 ) 3 − 4sin (3 ) = sin(3 )

Prin inducție se demonstrează că: = cos(3 ) = cos(3 arccos )

6. Să se determine formula termenului general al șirului dat de relațiile:

= ; ∈ [− , ]; = ( − ) ; ≥
Soluție: = ∈ [−1,1] ⇒ (∃) ∈ [0,2 ]; = cos ; = arccos

= (4 − 3) = cos (4cos − 3) = cos(3 )

= (4 − 3) = cos 3 (4cos 3 − 3) = cos(3 )

Prin inducție se demonstrează că: = cos(3 ) = cos(3 arccos )

7. Să se determine formula termenului general al șirului dat de relațiile:

= ; ∈ ℝ; = ; ≥

Soluție: = ∈ [−1,1] ⇒ (∃) ∈ − , ; = tg ; = arctg

( )
= = = tg(2 ) ; = = ( )
= tg(2 )

Prin inducție se demonstrează că: = tg(2 ) = tg(2 arctg )

8. Să se determine formula termenului general al șirului dat de relațiile:


= ; ∈ ℝ; = ; ≥

Soluție: = ∈ [−1,1] ⇒ (∃) ∈ − , ; = tg ; = arctg

( ) ( )
= = = tg(3 ) ; = = ( )
= tg(3 )

Prin inducție se arată că: = tg (3 ) = tg(3 arctg )

BIBLIOGRAFIE:
1. Gh. Sireţchi, “Calcul diferenţial şi integral”, Editura Ştiinţifică şi
Enciclopedică, Bucureşti, 1985
2. Daniel Sitaru,Claudia Nănuți, “Matematică pentru olimpiade”,
Editura Ecko-Print, Drobeta Turnu Severin, 2014

28 REVISTA DE MATEMATICĂ MEHEDINȚEANĂ NR. 15


S.S.M.ROMÂNIA - Filiala Mehedinți 2015

APLICAȚII ALE FUNCȚIEI ( + ), unde , ∈ (ℂ), ∈ℂ

Prof. Dan Nedeianu,Colegiul Național “Gheorghe Țițeica”


În cele ce urmează, ne propunem abordarea funcției polinomiale

( )= ( + )unde , sunt matrice din (ℂ), : ℂ → ℂ. Prin calcul direct, forma


algebrică a acestui polinom (evident de grad maxim ) va fi: ( ) = ( + )=
( ) + + + . Dacă , ∈ (ℝ), atunci { , , , } ⊂ ℝ, iar
dacă , ∈ (ℤ), atunci { , , , } ⊂ ℤ. Vom pune în evidență mai multe
aplicații (exemple) ale acestui rezultat, probleme apărute în diverse reviste școlare de
specialitate sau probleme de concurs ale olimpiadei de matematică.

Exemplul : Se consideră matricele , ∈ (ℂ) cu = − , = . Arătați că dacă


funcția polinomială definită prin ( ) = ( + ) are o rădăcină dublă,
atunci ( + )= . OJM 2010, Dan Nedeianu

Soluție:Folosind proprietățile matricei transpuse, avem ( ) = det( + ) =


= det(− + ) = (−1) det( − ) ⇒ ( ) = − (− ), ∀ ∈ ℂdeci funcție impară.
Avem ( ) = (det ) + + + det , ∀ ∈ ℂ. Din funcție impară ⇒ = 0 și
det = 0, iar ( ) devine ( ) = (det ) + cu = rădăcină dublă. Rezultă =
0, deci ( ) (
= det ) , ∀ ∈ ℂ. Se ajunge la 1) = det = det( + ).
(

Exemplul 2:Fie , , , numere reale și o rădăcină cubică nereală a unității. Arătați că


umătoarele afirmații sunt echivalente. 1. ( + )+ ( + )+ ( + )=
( + ) pentru orice matrice , ∈ (ℝ). 2. ∈ { , , }, = ,

= , = . G.M. 11/2013, Gabriel Necula, Ploiești

Soluție: Începem cu implicația 2 ⟹ 1, adică probăm că:

det( + ) + det( + ) + det( + ) = 3(det + det ), ∀ , ∈ (ℝ). Această


relație de demonstrat este o extensie a relației det( + ) + det( − ) = 2 det( + ),
valabilă pentru orice matrice , ∈ ( ℝ) .

Notăm ( ) = det( + ) = (det ) + + + det , : ℝ → ℝ cu , ∈ ℝ. Avem


(1) = det( + ) = det + + + det ,
( ) = det( + ) = det + + + det și

( ) = det( + ) = det + + + det . Prin sumare rezultă

(1) + ( ) + ( ) = 3 det + (1 + + ) + (1 + + ) + 3 det .

Întrucât 1 + + = 0 ⇒ relația de demonstrat. Reciproc, 1 ⟹ 2, presupunem că are loc


afirmația 1 și arătăm că , , sunt soluțiile ecuației = 1 și că = 3.

Luând ∈ (ℝ) nesingulară și = din 1⇒ 3 det = ∙ det ⇒

29 REVISTA DE MATEMATICĂ MEHEDINȚEANĂ NR. 15


S.S.M.ROMÂNIA - Filiala Mehedinți 2015
⇒ = 3. Se obține
det( + ) + det( + ) + det( + ) = 3(det + det ), ∀ ; ∈ (ℝ). Luând
= și ∈ ( )
ℝ nesingulară, din (*)⇒ ( + + ) det = 3 det , adică
+ + = 3. (1)

Apoi pentru = = se obține ( + 1) + ( + 1) + ( + 1) = 6.

Folosind (1) ⇒ + + + + + = 0. (2)

Pentru =− , = , din (*) se obține ( − 1) + ( − 1) + ( − 1) = 0

care conduce la + + = + + . (3).

Din (2) și (3) ⇒ + + = + + = 0. De aici + + = 0. Din relațiile


+ + = 0, + + = 0, + + = 0 ⇒

⇒ + + −3 =0⇒ = 1 ⇒ , , sunt soluțiile ecuației − 1 = 0, c.c.t.d.

Exemplul 3: Fie , , trei matrice de ordin care au ca elemente numere reale și care
îndeplinesc condițiile: ( + )= ( + ) și = = . Arătați că
( + )= ( + ). OJM 2009

Soluție: Putem nota ( ) = det( + ) = (det ) + + + det ,


( ) = det( + ) = (det ) + + + det , cu , : ℝ → ℝ, , , , ∈ ℝ

Avem: det( + ) = det( + ) ⇒ ( ) = ( ) ⇔ − det − + + det = det −


− det + = − det +
++ + det , de unde se ajunge la sistemul:
− + det = − + det

Dar det = det = det = ∈ℝ⇒ = și = .

Atunci det( + ) = (1) = det + + + det = det + + + det = ( 1) =

= det( + ), adică exact relația cerută.

Exemplul 4: Fie matricile , ∈ (ℤ) astfel încât = = ,

= . Să se arate că ∈ ℤ.

Lista scurtă ONM-2008, Dan Nedeianu, RMT- 4-2010

Soluție:Considerăm polinomul ( ) = det( + ) = (det ) + + + det . Din


enunț rezultă că ( ) = det( + ) = + , : ℝ → ℝ, , ∈ ℤ. Se observă că :

det( + ) = det( + ) ( − + ) = det( + ) ∙ det( + ) ∙ det( +


2 = 1 2, unde 3=1, ∈ℂ/ℝ .

Din (1) = + , ( ) ( )=( + )( + )= − + ⇒

30 REVISTA DE MATEMATICĂ MEHEDINȚEANĂ NR. 15


S.S.M.ROMÂNIA - Filiala Mehedinți 2015
⇒ det( + ) = + . Analog det( − ) = det( − ) det( − ) det( −
2 = 3− 3. Prin urmare det 3+ 3+det 3− 3=2 3 de unde rezultă cerința problemei.

Exemplul 5: Fie matricele , ∈ (ℝ) cu = . Să se arate că ( + )≥

OJM-2012

Soluție:Vom considera funcția polinomială ajutătoare ( ) = ( + + ), : ℂ →


ℂ cu forma algebrică ( ) = det( ) + + + det( + ) , , ∈ ℝ

Întrucât = ⇒ ( )= + + det( + ) (*)

Observăm că ( ) = det( + + ) = det[ + + ( − )] =

= det( + ) det( − ) și atunci (− ) = det( − ) det( + ) = ( ).

Revenind la (*)⇒ − − + det( + ) = − − + det( + ), de unde se deduce


că = 0. Pe de altă parte ( ) = det( + ) ∙ det( + ) = |det( + )| ≥ 0 adică
− + det( + ) ≥ 0 (1)

În fine (1) = det( + + + ) = det( + )( + ) = [det( + )] ≥ 0, însă

(1) = + det( + ) ≥ 0 (2) . Din (1) și (2) adunate ⇒ concluzia.

Exemplul 6: Fie matricea inversabilă de ordin cu elemente reale și funcția polinomială


( )= ( + ). Știind că polinomul are o rădăcină complexă nereală, să se arate
că toate rădăcinile sale au modulul egal cu .

G.M. 6-7-8/2012, Cosmin Manea, Petrică Dragoș, Pitești

Soluție: Avem det( − ) = det( − ) = det −( − ) = − det( − ) și astfel


det( − ) = 0, adică (−1) = 0. Fie ∈ ℂ/ℝ rădăcină pentru . Întrucât ( ) =
(det ) + + + det( ) are coeficienți reali și det = det( ) ≠ 0 rezultă că
( ) = 0 și din ultima relație Viete avem că ∙ ∙ (−1) = −1 ⇒ ∙ = 1 ⇒ |−1| = | | =
| | = 1.

Exemplul 7:Fie și două matrice pătratice de ordinul cu elemente reale cu


proprietățile: a) ( + (− ) ) = + (− ) , ∈ {− , }

b) = . Să se arate că ( + )=[ ( )+ ( )] .

GM 2/2013, Victor Marinescu, Craiova

Soluție: Fie funcția polinomială ( ) = det( + ) = det( ) + + + (det ) .


Conform ipotezei avem (1) = det( ) + det( ), iar
(−1) = det( ) − det( ) ceea ce înseamnă că det( ) + + + det( ) =

= det( ) + det( ) ⇒ + = 0 precum și

31 REVISTA DE MATEMATICĂ MEHEDINȚEANĂ NR. 15


S.S.M.ROMÂNIA - Filiala Mehedinți 2015
det( ) − + − det( ) = det( ) − det( ) ⇒ − = 0.

Astfel = = 0 ⇒ ( ) = det( ) + [det( )] . Prin urmare putem scrie:

det( + ) = det( + ) ∙ det( − + ) = det( + ) det( + ) det( + )


=

= (1) ( ) ( ) = [det( ) + det( )][det( ) + det( )]


[det( ) + det( )] = [det( ) + det( )] , c.c.t.d.

Exemplul 8: Fie , două matrice singulare de ordin cu elemente întregi astfel încât
= . Știind că ( + )= ( + ) = să se calculeze ( + ).

G.M. 2/2012, Dinu Șerbănescu, București

Soluție: Considerăm funcția : ℂ → ℂ, ( ) = det( + ). Cum det = det = 0, rezultă


( )= + . Avem 2 = det( + ) = (1) = + , iar 2 = det( + ) =
det( + ) ∙ det( − ) = ( ) ∙ (− ) = (− + )(− − ) = + unde =
−1, ∈ ℂ/ℝ.

Cum + = 2 și + = 2, obținem = = 1 ⇒ ( ) = det( + )= +

Mai departe, avem că: det( + ) = det( + ) ∙ det( − + )=

= det( + ) det( + ) det( + ),

ceea ce va implica: det( + ) = (1) ( ) ( ) = 2 ∙ ( + )( + )=

= 2( + ) = 2(−1) = 2

Exemplul 9: Fie ∈ (ℤ). Să se arate că divide determinantul matricei + , unde


este transpusa matricei . G.M. 5/2013, Marian Cucoaneș, Mărășești

Soluție: Fie = − ∈ (ℤ). Avem + = ⇒ =− ⇒ det = 0.

Pe același principiu, pentru , ∈ ℝ avem că

det( + )= det( ) + + + det( ) unde , ∈ ℤ.

Pentru = 5, = 5, în particular se va obține

det(3 + 2 ) = det(5 + 2 ) = 5 det + ∙5 ∙2+ ∙ 5 ∙ 2 , evident divizibil cu 5.

Exemplul 10: Fie , ∈ (ℝ) cu proprietatea că = și

( − + )− ( + + )+ = ( ).

Să se arate că ( − )= . G.M. 12/2012, Traian Tămîian, Carei


Soluție: Notăm ( ) = det( + ), ∈ ℂ și = − + , = 1.

32 REVISTA DE MATEMATICĂ MEHEDINȚEANĂ NR. 15


S.S.M.ROMÂNIA - Filiala Mehedinți 2015
Avem: ( )= + + + cu = det , det . Deoarece

= ⇒ det( − + ) = det( + ) ∙ det( + )=

= ( )∙ ( )=( + + + )( + + + ).

După calcule, folosind că: + + 1 = 0 ⇒ det( − + )=

=( + ) + + −( + ) −( + ) − (1)

Analog det( + + ) = det( − ) det( − )=

= (− ) ∙ (− ) = (− + − + )(− + − + )
După calcule, se ajunge la:

det( + + )=( − ) + + + ( − ) − ( − ) (2)

Relațiile (1) și (2), înlocuite în relația din enunț, conduc la


( + )( + ) = 1; însă , , , ∈ℤ⇒ + = + = ±1.

Avem det( − ) = (−1) = − + − = ( + )−( + )= 0

Exemplul 11: Se consideră matricele , ∈ (ℝ) cu = și ( )− ( )= .


Știind că ( + + ) = , să se calculeze ( − ). GM 6-7-8, Traian Tamâian

Soluție: Cum este firesc pornim de la funcția polinomială:

: ℂ → ℂ, ( ) = det( + ) = + + + , unde = det ,


= det , , , , ∈ ℝ și − = 1. Avem det( + + ) = det( − ) det( −
2 = − ∙ − 2, unde 3=1, ∈ℂ/ℝ..Ipoteza conduce la –
+ 2− + − + − 2+ =0. Deoarece − =1⇒ 2− +1 − 2+1=0.

După calcule, folosind că + + 1 = 0 se ajunge la + + − + = 0, adică


( + ) + ( − 1) + ( + 1) = 0. Întrucât , ∈ ℝ ⇒ + = − 1 = + 1 = 0 ⇒
= 1, = −1. Funcția polinomială devine ( ) = + − + .

De aceea det( − ) = (−1) = − + 1 + 1 + = − + 2 = 3.

PROBLEME ELEMENTARE TRATATE NEELEMENTAR

Prof.Daniel Sitaru, Colegiul Național Economic „Theodor Costescu”


Fără a intra în detalii privind teorema lui Fermat pentru funcții de mai multe variabile
,teoria formelor biliniare și formula Fubini pentru calculul integralelor duble și triple
putem considera cunoscute rezultatele acestora și le putem aplica la clasă sau la cercurile
de elevi la următoarele tipuri de probleme:

33 REVISTA DE MATEMATICĂ MEHEDINȚEANĂ NR. 15


S.S.M.ROMÂNIA - Filiala Mehedinți 2015

Problema 1:
Să se afle minimul expresiei:
( , )= + − − + ; , ∈ ℝ
Soluție:
E ʹ = 2x − 2 E ʹ = 0 x=1
ʹ ; ʹ ⟹
E = 2y − 4 E = 0 y=2

minE(x, y) = E(1,2) = 1 + 4 − 2 − 8 + 10 = 5
Problema 2
Să se afle minimul expresiei:
( , , )= + + − − + +
Soluție:
E ʹ = 2x − 2 E ʹ = 0 x=1
E ʹ = 2y − 4 ; E ʹ = 0 ; y = 2
E ʹ = 2z + 6 E ʹ = 0 z = −3
Problema 3
Pentru ce valoare > 0 , aria mulțimii:
= ( , ) ≤ ≤ ; ≤ ≤ +
este minimă? Se cere minimul ariei.
Soluție:

Fie : (0, ∞) → ℝ ;

⎛ ⎞ 3 3
( )= ( )= = ⎜ ⎟ = +
2
⎝ ⎠
ʹ(
9
)=0⟹ =1⟹ ( ) = ( 1) =
2
Problema 4
Să se arate că volumul cubului de latură “a” este = .
Soluție:
Fie ( , , ) = {( , , )|0 ≤ ≤ ,0 ≤ ≤ , 0 ≤ ≤ }.

= = = =

= =

Problema 5
Să se arate că volumul paralelipipedului dreptunghic de laturi , , este = .
Soluție:
Fie ( , , ) = {( , , )|0 ≤ ≤ ,0 ≤ ≤ , 0 ≤ ≤ }.

34 REVISTA DE MATEMATICĂ MEHEDINȚEANĂ NR. 15


S.S.M.ROMÂNIA - Filiala Mehedinți 2015

= = = =

= =

Problema 6
Să se arate că volumul prismei patrulatere regulate cu latura bazei „a” și înălțimea “h”
este = .
Soluție:
Fie ( , , ) = {( , , )|0 ≤ ≤ ,0 ≤ ≤ , 0 ≤ ≤ ℎ}.

= = = ℎ =

= ℎ = ℎ

Problema 7
Să se demonstreze că aria laterală și volumul cilindrului circular drept de rază „r” și
înălțime „h” sunt date de formulele:
= ; =
Soluție:

: [0, ℎ] → ℝ; ( ) = ; ʹ( ) = 0

( ) ℎ
= = = = ℎ
0

( ) 1+ ʹ ( ) ℎ
=2 =2 1+0 =2 =2 ℎ=2
0
Problema 8

Să se demonstreze că aria laterală și volumul conului circular drept de rază „r” , înălțime
„h”, generatoare „g” sunt date de formulele:

= ; =

35 REVISTA DE MATEMATICĂ MEHEDINȚEANĂ NR. 15


S.S.M.ROMÂNIA - Filiala Mehedinți 2015
Soluție:

1
: 0 0 1 =0; : ℎ− = 0; : = ; : [0, ℎ] → ℝ; ( ) =
ℎ 1
ʹ( ʹ
ℎ +
)= ;1 + ( )=1+ = =
ℎ ℎ ℎ ℎ
ℎ ℎ ℎ
= ( ) = = ∙ = ∙ =
ℎ ℎ 3 0 ℎ 3 3

2 ℎ
=2 ( ) 1+ ʹ ( ) =2 ∙ = ∙ ∙ =
ℎ ℎ ℎ ℎ 2 0

Problema 9
Să se demonstreze că aria laterală și volumul trunchiului de con circular drept de raze „R,
r” , înălțime „h”, generatoare „g” sunt date de formulele:
= ( + ) ; = ( + + )

Soluție:

1
: 0 1 =0
ℎ 1

36 REVISTA DE MATEMATICĂ MEHEDINȚEANĂ NR. 15


S.S.M.ROMÂNIA - Filiala Mehedinți 2015

: + ℎ−ℎ − =0; : ℎ = ( − )+ℎ ; : = +


ʹ( ʹ ( )
: [0, ℎ] → ℝ; ( ) = + ; )= ;1 + ( )= 1+ =
− 2( − )
= ( ) = + + =
ℎ ℎ
( − ) ℎ 2 ( − ) ℎ ℎ ( − ) ℎ
= ∙ + ∙ + = + ( − ) ℎ+ ℎ
ℎ 3 0 ℎ 2 0 0 3
=
ℎ ℎ
= ( −2 + +3 −3 +3 ) = ( + + )
3 3

=2 ( ) 1+ ʹ ( ) =2 + ∙ =
ℎ ℎ
2 − ℎ ℎ 2 − ℎ −
= ∙ + = ∙ + ℎ =2 + = ( + )
ℎ ℎ 2 0 0 ℎ ℎ 2 ℎ
Problema 10
Să se demonstreze că aria și volumul sferei de rază „R” sunt date de formulele:

= ; =
Soluție:

ʹ( ʹ
: [− , ] → ℝ; ( ) = √ − ; )=

; 1 + ( )=

= ( ) = ( − ) = − =
− 3
2 4
= ( + )− ( − (− ) ) = 2 − =
3 3 3

=2 ( ) 1+ ʹ ( ) =2 − ∙ =
√ −
=2 =2 − (− ) = 4

37 REVISTA DE MATEMATICĂ MEHEDINȚEANĂ NR. 15


S.S.M.ROMÂNIA - Filiala Mehedinți 2015

Problema 11
Să se demonstreze că lungimea cercului de rază „R” este dată de formula:
=
Soluție:

ʹ
: [− , ] → ℝ; ( ) = √ − ; 1+ ( )=1+ =

=∫ 1+ ʹ ( ) =∫ = arcsin = (arcsin 1 − arcsin (−1)) ==


√ −
+ = ; = ⇒ =2

Problema 12
Să se demonstreze că aria discului de rază „R” este dată de formula:
=
Soluție:
=∫ ( ) =∫ √ − = ; =

BIBLIOGRAFIE:
1. C. Năstăsescu, C.Niță , Gh.Rizescu –“Algebra”-Manual pentru clasa a
IX-a-Editura Didactică și Pedagogică-București-1997
2. Daniel Sitaru,Claudia Nănuți-“Matematici pentru olimpiade”–Editura
Ecko-Print-Drobeta Turnu Severin-2014
PROBLEME PROPUSE

Elevii pot rezolva probleme de la clasa pe care o urmează, de la clasa inferioară precum și de la
orice clasa superioară. Soluţiile redactate pe foi format A4 se vor preda profesorului îndrumător
care va transmite punctajele elevilor colectivului de redacție.
Clasa I
I.1. Andrei a scăzut două numere mai mici decât 7 şi a obţinut 6. Care sunt cele două numere?

Prof. Mariana Cornea


I.2. Gigel a numerotat paginile unui caiet de la 1 la 15. Câte cifre a folosit?

Prof. Mariana Cornea

38 REVISTA DE MATEMATICĂ MEHEDINȚEANĂ NR. 15


S.S.M.ROMÂNIA - Filiala Mehedinți 2015

I.3. Într-o cutie sunt bile albe și albastre. Dacă oricum am extrage 6 bile, vom avea bile de ambele
culori,arătați că nu pot fi mai mult de 10 bile în cutie. Prof. Maria Ungureanu

I.4. Mă gândesc la un număr. Scad din el răsturnatul numărului 63, apoi vecinul mai mic al
numărului 29. La rezultatul obţinut adaug cel mai mic număr de zeci şi obţin un număr mai mic
decât 17. La ce număr m-am gândit? Prof. Gheorghița Tufiș

I.5. Dacă dintr-o oră: un sfert de oră pictez, 5 minute mănânc un măr şi 3 minute îmi pregătesc
cărţile ,cât timp mai am să mă relaxez? Prof. Gheorghița Tufiș

I.6. Ionel are 34 lei iar sora 56 lei. Ionel vrea să aibă o sumă cu 4 lei mai mare decât sora sa. De cât
ar mai avea nevoie? Prof. Gheorghița Tufiș

I.7. La ora de educaţie fizică elevii clasei I sunt aşezaţi în rând, unul lâgă altul. Andreea observă că
de oriunde s-ar număra (din dreapta ori din stânga) ea este a treisprezecea. Câţi elevi sunt in clasa
I? Prof. Gheorghița Tufiș

I.8. Cu câte litere se scrie de două ori cuvântul „matematica”? Prof. Maria Ungureanu

Clasa a II-a
II.1. Într-un colaj s-au folosit 19 cerculețe, ceea ce înseamnă mai mult cu 3 decât triunghiuri și mai
puțin cu 4 decât pătrate. Câte figuri s-au folosit? Prof. Mariana Cornea
II.2. Dintr-un număr, scăzându-l pe 26, se obţine un rezultat care este cu 7 mai mare decât cel mai
mare număr par scris cu o cifră. Află numărul şi verifică relaţiile. Prof. Mariana Cornea

II.3. Câte numere de forma nu sunt impare şi au suma cifrelor 12? Prof. Gheorghița Tufiș

II.4. Tudor doreşte să cumpere 4 cărţi, dar ar mai avea nevoie de 36 lei. Dacă ar cumpăra doar 3
cărți, ar rămâne cu 23 de lei. Câţi lei are Tudor? Prof. Gheorghița Tufiș

II.5. O familie are mai mulţi copii: fete şi băieţi. Care este numărul cel mai mic de copii pentru ca
fiecare dintre ei să aibă cel puţin câte două surori şi doi fraţi? Prof. Gheorghița Tufiș

II.6. De câte ori foloseşti cifra 3 pentru a scrie numerele de la 220 până la 243?

Prof. Gheorghița Tufiș

II.7. Scrieți toate numerele naturale cuprinse între și care se împart exact la .

Prof. Maria Ungureanu

II.8. Scrieți toate numerele pare, cuprinse între și și micșorate cu .

Prof. Maria Ungureanu

II.9. Calculați: ( ∙ + ∙ + ∙ + ∙ ): ( + + + ) Prof. Maria Ungureanu

II.10. Determinați ultima cifră a numărului: + ∙ + ∙ ∙ + ∙ ∙ ∙ + ∙ ∙ ∙ ∙

Prof. Maria Ungureanu

39 REVISTA DE MATEMATICĂ MEHEDINȚEANĂ NR. 15


S.S.M.ROMÂNIA - Filiala Mehedinți 2015

Clasa a III-a
III.1. O față de masă are forma de dreptunghi cu suma laturilor de 120 cm. Dacă este pliată pe
jumătate are forma de pătrat.Care sunt dimensiunile feței de masă? Prof. Maria Ungureanu

III.2. De câte ori se folosește cifra 5 pentru numerotarea unei cărți care are 105 pagini?

Prof. Maria Ungureanu

III.3. Suma a trei numere este 322. Dacă din primul scad 13, din al doilea 24 şi din al treilea 50 al
doilea va fi cu 60 mai mare decât dublul primului şi cu 10 mai mic decât al treilea. Ce valoare are
cel mai mare număr? Prof. Gheorghița Tufiș

III.4. Mărind cu 7 triplul unui număr natural obţinem un număr cu 33 mai mare decât numărul
iniţial. Care este numărul? Prof. Gheorghița Tufiș

III.5. Suma a cinci numere este 81. Primele trei sunt consecutive impare. Câtul dintre al patrulea şi
al cincilea este 2 iar suma lor este 60. Să se afle diferenţa dintre ultimul şi primul număr.

Prof. Gheorghița Tufiș

III.6. La o florărie s-au primit 50 de lalele roşii, galbene şi violet. Câte lalele roşii s-au adus ştiind că
la 3 lalele galbene s-au primit 5 lalele roşii şi două lalele violet? Prof. Gheorghița Tufiș

III.7. Suma a trei numere pare consecutive este 186. Aflați numerele. Prof. Dorel Căpraru

Clasa a IV-a
IV.1. Suma a trei numere naturale este 72. Aflați numerele, știind că primul număr este de 3 ori
mai mic decât suma celorlalte două, iar diferența dintre al treilea și al doilea este de 9 ori mai mică
decât primul număr. Prof. Maria Ungureanu

IV.2. Într-o cofetărie dacă stau câte 2 persoane la o masă rămân 10 persoane în picioare, iar dacă
stau câte 3 persoane la o masă, rămân 2 mese libere. Câte mese și câte persoane sunt în cofetărie?

Prof. Maria Ungureanu

IV.3. La un concurs de matematică s-au propus spre rezolvare 10 probleme. Pentru fiecare
problemă rezolvată bine s-au acordat 5 puncte, iar pentru fiecare probleme nerezolvată s-au scăzut
3 puncte. Câte probleme au fost rezolvate bine de către un elev care a obţinut 34 de puncte ?

Prof. Gheorghița Tufiș

IV.4. Ce oră este dacă mai rămân din zi 5/3 ore din câte au trecut? (Ziua se consideră de 24 de ore
şi începe la ora 12 noaptea). Prof. Gheorghița Tufiș

IV.5. O mamă cu 2 copii au împreună 60 de ani. Să se afle vârsta fiecăruia dintre copii, ştiind că cel
mare are de 3 ori vârsta celui mic şi că mama are dublul sumei vârstelor copiilor.

Prof. Gheorghița Tufiș

IV.6. Alin îi zice lui George: dă-mi o pară şi voi avea îndoit cât vei avea tu. George răspunde: dă-mi
tu o pară şi voi avea şi eu cât tine. Câte pere avea fiecare? Prof. Gheorghița Tufiș
40 REVISTA DE MATEMATICĂ MEHEDINȚEANĂ NR. 15
S.S.M.ROMÂNIA - Filiala Mehedinți 2015

IV.7. Albă ca Zăpada și fiecare dintre cei șapte pitici au trimis câte un cadou celorlalți. Câte cadouri
s-au trimis în total? Prof. Piț-Rada Marica
IV.8. Fie numărul N=123456...20142015, obținut prin lipirea , în ordine crescătoare, a primelor
2015 numere naturale nenule. Aflați câte cifre s-au folosit pentru a scrie numărul N.
Prof. Piț-Rada Marica
IV.9. Pentru un spectacol în aer liber se poate pune un număr de scaune în rânduri de câte sau

de câte . Știind că diferența dintre numărul de rânduri, în cele două cazuri este să se afle
numărul de scaune. Prof. Maria Ungureanu

IV.10. Care este cel mai mic număr natural nenul care se împarte exact la , și ?

Prof. Dorel Căpraru

Clasa a V-a
V.1. Aflați numerele prime și pentru care + si ∙ + sunt numere prime.

Prof. Constantin Ionică

V.2. Fie un număr natural și ( ) suma cifrelor sale.

a. Să se arate că − ( ) este divizibil cu .


b. Să se arate că dacă ( ) = ( ) atunci este divizibil cu .
Prof. Daniel Sitaru

V.3. Fie un numӑr prim și o mulțime formatӑ din numere naturale consecutive. Sӑ se arate
cӑ nu existӑ douӑ mulțimi și , cu = ∪ , astfel încât produsul elementelor mulțimii sӑ
fie egal cu produsul elementelor mulțimii . Prof. Elena Ghilerdea

V.4. Se cere câtul împărțirii sumei tuturor numerelor de forma la .

Prof. Claudia Nănuți


V.5. Arӑtați cӑ nu existӑ numere naturale astfel încât : !+ !+⋯+ ! = unde
!= ∙ ∙ …∙ Prof. Elena Ghilerdea

V.6. Să se arate că oricare ar fi ∈ ℕ∗ există , , , , ∈ ℕ încât:

+ = + + + Prof. Daniel Sitaru

V.7. Să se demonstreze că numărul: … nu este prim. Prof. Daniel Sitaru


" "

V.8. Se consideră mulțimea ={ ∙ | , ∈ ℕ}. Să se arate că :

a. , ∈ , b.∀ , ∈ ⟹ ∙ ∈ . Prof. Ovidiu Ticuși

V.9. Să se arate că numărul = + este divizibil cu . Se cer câturile împărțirii lui la


și la . Prof. Mihai Octavian Ungureanu

41 REVISTA DE MATEMATICĂ MEHEDINȚEANĂ NR. 15


S.S.M.ROMÂNIA - Filiala Mehedinți 2015
V.10. Să se arate că numărul = − − + este divizibil cu .

Prof. Mihai Octavian Ungureanu

V.11. Arătați că există două numere dintre , , , , , care prin împărțirea la dau
același rest. Aflați numerele cu această proprietate. Prof. Gabriela Bondoc

V.12. Să se determine ∈ ℕ astfel încât = + + . Prof. Ovidiu Ticuși

V.13. Fie , ∈ ℤ astfel încât: + . Se cere restul împărțirii lui + la .

Prof. Daniel Sitaru

V.14. Să se arate că numărul: + + ⋯+ este divizibil cu .

Prof. Claudia Nănuți


V.15. Să se compare numerele și . Prof. Daniel Sitaru

V.16. Suma tuturor pătratelor perfecte de cifre poate fi pătrat perfect? Justificați răspunsul.

Prof. Ecaterina Pupăză

V.17. Un număr natural se poate scrie ca suma a două pătrate perfecte. Dublul său și pătratul său
au aceeași proprietate? Prof. Ecaterina Pupăză

V.18. Să se determine , , ∈ ℕ încât: , ( )+ , ( )+ , ( ) = , ( ).

Prof. Daniel Sitaru

V.19. Fie , , numere prime distincte mai mari decât . Se cere restul împărțirii sumei
+ + la . Prof. Daniel Sitaru

V.20. Să se rezolve ecuația: + + + = + + + ( + + )

Prof. Daniel Sitaru

V.21. Determinați cifrele , , astfel încât + = +

Prof. Elena Rîmniceanu

V.22. Produsul a două numere naturale consecutive este mai mic decât dublul numărului cu
. Să se determine cele două numere știind că : = + + +⋯+

Prof. Elena Rîmniceanu

V.23. Se numește număr natural stabil un număr natural în care orice două cifre alăturate au suma
mai mare decât 9. Dacă un număr natural nu este stabil , atunci putem aplica operații de înlocuire,
a oricăror două cifre alăturate care au suma mai mică sau egală cu 9, cu o singură cifră cu valoarea
egală cu suma celor două cifre alăturate. De exemplu din 2353 putem obține 283 sau 58 , ambele
fiind stabile. Se cere să se determine care este cel mai mare număr stabil care se poate obține din
324532. Prof. Piț-Rada Ionel-Vasile

42 REVISTA DE MATEMATICĂ MEHEDINȚEANĂ NR. 15


S.S.M.ROMÂNIA - Filiala Mehedinți 2015
V.24. Este posibil ca raportul a două numere prime să fie egal cu media lor aritmetică?

Prof. Emilia Răducan

V.25. Scrieți numărul ca diferență a două pătrate perfecte. Prof. Elena Rîmniceanu

V.26. Fie un număr natural cu ultima cifră . În produsul ! se elimină toți factorii pari și cei
divizibili cu 5. Care este ultima cifră a produsului după aceste eliminări?

Prof.dr. Gheorghe Căiniceanu

V.27. Să se afle știind că = . Prof. Marica Ștefan

V.28. Să se afle știind că =( + + ) . Prof. Marica Ștefan

V.29. Să se arate că în șirul ( ) ∈ℕ , = + , există o infinitate de perechi de termeni


consecutivi cu proprietatea ca suma lor este un pătrat perfect. Prof. Dana Paponiu

V.30. Să se determine cifrele , , astfel încât: = = = . Prof. Daniel Sitaru

V.31. Fie ∈ ℕ∗ , ≥ ℕ .Să se determine , , astfel încât:

( ) + ( ) + ( ) = + + Prof. Daniel Sitaru

V.32. Câte numere de forma sunt divizibile cu și au proprietatea că este pătrat


perfect? Prof. Daniel Sitaru

V.33. Aflați , , , ∈ ℕ încât: ∙ = . Prof. Daniel Sitaru

V.34. Un triunghi are măsurile unghiurilor proporționale cu , , . Să se arate că triunghiul


are un unghi de °. Generalizare pentru măsurile unghiurilor numere naturale consecutive.

Prof. Daniel Sitaru

Clasa a VI-a
VI.1. Arătați că numărul = | − |+| − |+ | − |+⋯+| − | este divizibil cu
, ∀ ≥ . Prof. Bălășoiu Daniela

VI.2. Determinați numerele , știind că numerele , ș sunt direct proporționale cu 3,


14 și 7 , iar suma + + = . Prof. Bălășoiu Daniela

VI.3. Să se arate că: = − + − + ⋯+ − nu este număr natural.

Prof. Daniel Sitaru

VI.4. Demonstrați cӑ nu existӑ nici un numӑr natural de forma 2n scris cu toate cifrele luate o
singurӑ datӑ. Prof. Elena Ghilerdea

VI.5. Să se determine restul împărțirii lui: = + + la 403.

43 REVISTA DE MATEMATICĂ MEHEDINȚEANĂ NR. 15


S.S.M.ROMÂNIA - Filiala Mehedinți 2015
Generalizare: Fie , , ∈ ℕ∗ . Să se determine restul împărțirii lui:

=( − ) +( + ) + la . Să se determine restul împărțirii lui:

=( − ) +( + ) + la . Prof. Daniel Sitaru

VI.6. Să se arate că numărul: = + nu poate fi scris ca sumă de două numere prime.

Prof. Daniel Sitaru


°
VI.7. Se știe că ∙
+

+⋯+ ( )( )
= . Pornind de la măsura unghiului de soluție a
°
ecuației precedente construiți un unghi de măsura . Prof. Alexandru Toporan

VI.8. Am câteva creioane și am constatat cӑ: 1.toate creioanele, mai puțin 4,sunt verzi;2.toate
creioanele, mai puțin 3, sunt roșii;3.toate creioanele, mai puțin 5, sunt albastre.Dacӑ am astfel de
creioane, câte creioane de altӑ culoare decât roșu, verde sau albastru am?

Prof. Elena Ghilerdea


!
VI.9. Să se demonstreze că fracția: este ireductibilă. ( ! = ∙ ∙ … ∙ ; ∈ ℕ∗ )
!

Prof. Daniel Sitaru

VI.10. Determinați , ∈ ℕ astfel încât: = ( + + ) Prof. Mihai Octavian Ungureanu

VI.11. Arătați că + +⋯+ < . Prof. Alexandru Toporan

VI.12. La un concurs de matematică s-au dat 15 probleme. Pentru fiecare problemă rezolvată
corect se primesc 7 puncte, pentru una rezolvată greșit se scad 4 puncte, iar pentru una
neabordată se acordă zero puncte. Câte probleme a rezolvat corect un elev dacă el a obținut 11
puncte? Prof. Constantin Ionică

VI.13. Fie . , ∈ ℕ, ≠ ≠ ≠ . Arӑtați cӑ ( + )( + )( + ) este un numӑr par.

Prof. Elena Ghilerdea

VI.14. Măsurile unghiurilor formate în jurul unui punct sunt exprimate în grade prin puteri ale
lui sau . Aflați numărul minim de unghiuri în condițiile date. Prof. Alexandru Toporan

VI.15. Fie , ∈ ℕ∗ . Dacă: + + + ⋯+ = arătați că: | −


Prof.Daniel Sitaru

VI.16. Se dau de unghiuri în jurul unui punct astfel ca ∢( ) = ° , ∢( )=


° ( ) ° ( ) °
,…, ∢ = iar ∢ = . Arătați că printre semidreptele ce
formează unghiurile există ce formează unghi alungit. Prof. Alexandru Toporan

VI.17. Pe segmentul alegem punctele , ,…, astfel încât = , = ,


= și așa mai departe = . Lungimile segmentelor se exprimă prin numere
naturale. Dacă = !− ∙ ! Aflați cea mai mare valoare a lui precum și
lungimea segmentului . S-a notat ! = ∙ ∙ ∙ … ∙ . Prof. Alexandru Toporan
44 REVISTA DE MATEMATICĂ MEHEDINȚEANĂ NR. 15
S.S.M.ROMÂNIA - Filiala Mehedinți 2015

VI.18. Ce număr trebuie adăugat la dreapta numărului trei cifre astfel încât numărul obținut
să se împartă cu , , ? . Prof. Alexandru Toporan

VI.19. Să se determine numerele , și știind că = = și

+ + = . Prof. Alexandru Toporan

VI.20. a.Arătați că − ⋮ b. Să se găsească numerele naturale și dacă


+ + = . c. Arătați că are loc inegalitatea:

+ + + ⋯+ < . Prof. Gabriela Bondoc

VI.21. Numărul împărțit la dă restul și împărțit la dă restul . Se cere restul împărțirii lui
la . Prof.Daniel Sitaru

VI.22. Fie = un segment pe care considerăm punctele , ,…, ; ∈ ℕ∗ încât:

= ; = ; = ;…; =

Aflați un număr ∈ ℕ∗ încât < 1 . ( Angstrom = ). Prof.Daniel Sitaru

VI.23. Se consideră un triunghi oarecare având cea mai mare latură de cm. Să se arate că oricum
am alege puncte în interiorul sau pe laturile triunghiului există două dintre ele aflate la o
distanță de cel mult mm unul de altul. Prof.Daniel Sitaru

VI.24. Determinați restul împărțirii numărului n=431999...99 la 27, unde numărul n are la sfârșit
2015 cifre de 9. Prof. Piț-Rada Marica
VI.25. Fie S suma divizorilor naturali ai numărului 2015. Să se arate că 42∙S este un număr natural
pătrat perfect. Prof. Piț-Rada Marica
VI.26. Aflați cel mai mic număr natural care are exact 42 divizori. Prof. Piț-Rada Marica
VI.27. Fie > 1 și ∈ ℕ∗ ; > 2. Să se arate că: + + +⋯+ > .

Prof.Daniel Sitaru

VI.28. Demonstrați că: + + ⋯+ < . Prof. Elena Rîmniceanu


∙ ∙ ∙ ∙ ∙ ∙

VI.29. Să se arate că numerele + și + ∙ sunt prime între ele.

Prof. Elena Rîmniceanu

VI.30. Să se afle numerele : − ; − ; ; + ; + ; + ; +

știind că toate sunt numere prime. Prof. Marica Ștefan

VI.31. Să se afle știind că: − = . Prof. Marica Ștefan

VI.32. Să se determine numerele naturale , , încât: − − − = .

Prof. Dana Paponiu

45 REVISTA DE MATEMATICĂ MEHEDINȚEANĂ NR. 15


S.S.M.ROMÂNIA - Filiala Mehedinți 2015

VI.33. Să se arate că dacă + + și + + sunt multiplii de ; , , ∈ ℕ atunci și


+ + este multiplu de . Prof. Dana Paponiu

Clasa a VII-a
VII.1. Să se determine raza cercului înscris în triunghiul de laturi , , știind că:

− + +| − |= − − . Prof. Mihai Octavian Ungureanu

VII.2. Determinați măsura unui unghi știind că măsura suplementului său este egală cu măsura
complementului zecimii sale. Prof. Mihai Octavian Ungureanu

VII.3. Să se rezolve ecuația: + + +⋯+ = Prof.Dan Nănuți

VII.4. Fie = … . Să se arate că √ este număr irațional.

Prof. Daniel Sitaru

VII.5. Pe latura [ ] a triunghiului oarecare se construiește în exteriorul său pătratul .


Dacă ∩ = { }, ∩ = { } si ⊥ , ∈ ( ), ⊥ , ∈ ( ), demonstrați
că patrulaterul este pătrat. Prof. Constantin Ionică

VII.6. Să se rezolve în ℕ ecuația: + − = ∙ . Prof. Daniel Sitaru

VII.7. Arătați că dacă = ∙ ∙ atunci √ ∈ ℝ ∖ ℚ.

Prof. Alexandru Toporan

VII.8. Fie un patrulater convex, astfel încât bisectoarea unghiului este paralelă cu .

Bisectoarea unghiului se intersectează cu în și cu în . Să se arate că dacă =


atunci este bisectoarea unghiului ∢ . Prof. Alexandru Toporan

VII.9. Fie = + + +⋯+



∙ . Pentru ce valoare a lui , are
divizori? Prof. Alexandru Toporan

VII.10. Numerele naturale se scriu sub forma următoare:



……………………
Pe ce linie se află numărul ? Ce număr ocupă penultima poziție în linia ?

Prof. Alexandru Toporan

VII.11. Se dă triunghiul . Fie ∈[ ] și ∈[ ], astfel încât = . Dacă


= ∩ și = . Arătați că ∥ . Prof. Alexandru Toporan
{[ , ] ( , )}
VII.12. Arătați că ≥√ , > , > . Prof. Alexandru Toporan

46 REVISTA DE MATEMATICĂ MEHEDINȚEANĂ NR. 15


S.S.M.ROMÂNIA - Filiala Mehedinți 2015

VII.13. Să se arate că dacă , , , ∈ ℚ și + √ = + √ atunci = și = .

Prof. Claudia Nănuți


VII.14. Să se arate că dacă , , , , , , , ∈ ( , ∞)

√ + +√ +√ = ( + + + )( + + + )

atunci , , , sunt direct proporționale cu , , , . Prof. Daniel Sitaru

VII.15. Să se arate că dacă , , ∈ ( , ∞) atunci:

+ + + + + + ≥ Prof. Daniel Sitaru

VII.16. Să se arate că dacă , , , , ∈ ℝ, + + + + ≥ atunci:

+ + + + ≥ . Prof. Daniel Sitaru

VII.17. Fie , , ∈ ℝ; + + = . Să se arate că:

− + − + − ≥√ ( + + )

Prof. Claudia Nănuți


VII.18. Fie , , , ∈ ℝ; + + + = . Să se arate că:

− + − + − + − ≥√ ( + + + )

Prof. Daniel Sitaru



VII.19. Dacă = , atunci media geometrică a numerelor și este un număr

natural. Prof. Elena Rîmniceanu

VII.20. Arătați că: + + + ⋯+ >√ Prof. Piț-Rada Marica


√ √ √
VII.21. Se scriu pe tablă numerele , , și apoi se înlocuiesc cu mediile aritmetice ale lor, luate
două câte două.Procedeul se repetă. Putem obține după mai mulți pași numerele

, ; ( ); , ( )? Prof. Emilia Răducan

VII.22. Fie , numere pozitive + = . Să se arate că: √ + +√ + < 4.

Prof. Emilia Răducan

VII.23. Fie , , numere pozitive + + = . Să se arate că:

√ + +√ + +√ + < Prof. Emilia Răducan

VII.24. Să se determine un număr astfel încât + = . Prof. Claudia Nănuți

VII.25. Arătați că numărul: este număr natural.

Prof. Elena Rîmniceanu

47 REVISTA DE MATEMATICĂ MEHEDINȚEANĂ NR. 15


S.S.M.ROMÂNIA - Filiala Mehedinți 2015

VII.26. Arătați că + + ∈ ℚ, ∀ ∈ ℕ. Prof. Elena Rîmniceanu

VII.27. Să se afle două numere cu proprietatea că există , ∈ ℕ încât:

∙ + ∙ = + . Prof. Marica Ștefan

VII.28. Să se afle aria paralelogramului știind că = ; = √ ; = .

Prof. Marica Ștefan

VII.29. Să se afle , , ∈ ℕ încât: − + + − + ∈ℕ

Prof. Marica Ștefan

VII.30. Câte soluții are următoarea ecuație în mulțimea numerelor întregi:

+ + + + + + = . Prof. Marica Ștefan

VII.31. Fie > 0 și ∈ ℕ. Să se arate că: + + > + . Prof. Daniel Sitaru

VII.32. Fie ABCD un patrulater convex cu AB,CD neparalele , AD, BC neparalele și o dreaptă d care
nu trece prin A, B, C sau D și care intersectează AB, DC, AD, BC în M, N, P respectiv Q. Demonstrați
MA QB NC PD
că : MB ∙ QC · ND · PA = 1. Elev Șeitan Radu Cătălin

Clasa a VIII-a

VIII.1. Să se simplifice expresia: ( )= . Prof. Ion Nănuți

VIII.2. Să se afle raportul dintre aria sferei înscrise și aria sferei circumscrise unui tetraedru regulat.
Prof. Mihai Octavian Ungureanu

VIII.3. Fie , , , ∈ ( , ∞) . Să se arate că:

+ + + + ≥

Prof. Daniel Sitaru

VIII.4. Fie o piramidă patrulateră regulată cu toate muchiile egale cu cm. Se cere
volumul sferei circumscrise piramidei. Prof. Mihai Octavian Ungureanu

VIII.5. Fie , , , puncte necoplanare astfel încât este echilateral și

(≮ )= (≮ )= °. Demonstrați că ( )⊥( ). Prof. Constantin Ionică

VIII.6. Să se determine , , ∈ ℕ∗ astfel încât: + + = + + .

Prof. Daniel Sitaru


48 REVISTA DE MATEMATICĂ MEHEDINȚEANĂ NR. 15
S.S.M.ROMÂNIA - Filiala Mehedinți 2015
VIII.7. Fie , , , , , ∈ ( , ∞) încât: = = = . Să se arate că:

( + )( + )( + )( + )( + )( + ) ≥

Prof. Daniel Sitaru

VIII.8. Arătați că numărul :N =(2016 - √ ⋯


)2015 : 2015 , este pătrat perfect.

Prof. Bălășoiu Daniela

VIII.9. Arătați că numărul


a= + √ + ⋅ este natural.

Prof. Bălășoiu Daniela

VIII.10. Fie o piramidă triunghiulară regulată în care latura bazei este = și înălțimea
piramidei este − ; ∈ ( , ). Să se arate că dacă = atunci piramida are volumul maxim.

Prof. Daniel Sitaru

VIII.11. Într-un paralelipiped dreptunghic cu lungimile muchiilor , , , diagonala și volumul


avem: √ + +√ = . Să se arate că: ≥ √ . Prof. Daniel Sitaru

VIII.12. Să se afle volumul sferei înscrise și volumul sferei circumscrise conului de rază și

înălțime 8 . Prof. Dorel Căpraru

VIII.13. Dacӑ , , ∈ ℝ∗ astfel încât : + + = și + + = , arӑtați cӑ


numӑrul = − − − este pӑtrat perfect. Prof. Elena Ghilerdea

VIII.14. Fie , , , , , ∈ ( , ∞) astfel încât: = = = . Să se arate că:

( + )( + )( + )( + )( + )( + ) ≥ . Prof. Daniel Sitaru

VIII.15. Fie , , , , , ∈ ( , ∞). Să se arate că: ( )( )( )


≤ + .

Prof. Daniel Sitaru

VIII.16. Să se arate că dacă ∈ ℕ atunci: ( + )( + )( + )+

este pătrat perfect. Prof. Daniel Sitaru

VIII.17. Să se arate că în orice triunghi avem relația: ( )


+( )
+( )

Prof. Daniel Sitaru

49 REVISTA DE MATEMATICĂ MEHEDINȚEANĂ NR. 15


S.S.M.ROMÂNIA - Filiala Mehedinți 2015
VIII.18. Fie ∈ ℕ∗ ; ∈ ℝ, , ,…, ∈ ( , ∞). Să se arate că:
+ +⋯+ + + +⋯+ + + + ⋯+
+ + ⋯+ ≥ ( + − )

Prof. Daniel Sitaru

VIII.19. O piramidă triunghiulară cu toate muchiile egale, are suma lungimilor muchiilor de
cm. Ce lungime are apotema sa? Prof. Elena Rîmniceanu

VIII.20. Să se arate că soluția ecuației √ − √ + = − ( − )+


este un număr natural. Prof. Elena Rîmniceanu

VIII.21. Se dă funcția : ℝ → ℝ, ( ) = − . Să se determine punctul de pe grafic în care


abscisa este egală cu ordonata, știind că graficul funcției trece prin punctul ( ; ).

Prof. Elena Rîmniceanu

VIII.22. Dacă + = , ≠ atunci aflați numărul = + + + + + + + .

Prof. Piț-Rada Marica


∗ | |
VIII.23. Funcția : ℝ ∖ { } → ℝ verifică relația: = −

Să se rezolve ecuația: [ ( )] = , unde [ ] − reprezintă partea întreagă a numărului real .

Prof. Daniel Sitaru

VIII.24. Să se rezolve ecuația: =− , unde { } − reprezintă partea fracționară a


numărului real . Prof. Daniel Sitaru

VIII.25. Fie , ,…, ∈ ( , ∞); + + ⋯+ = , ∈ ℕ; ≥ . Să se arate că:

( + )
( + )+ ( + )+ ⋯+ ( + )≤

Prof. Daniel Sitaru

VIII.26. Fie ( ) = + + ; a. Arătați că ( ) > , oricare ar fi real;

b. Aflați ( ) și valoarea lui pentru care se obține. Prof. Elena Rîmniceanu

VIII.27. Fie = { | ∈ ℤ ș + ∈ [− , ]}.Aflați media aritmetică a elementelor mulțimii .

Prof. Elena Rîmniceanu

√ √ ∙ √ √
VIII.28. Arătați că numărul: este număr natural.

Prof. Elena Rîmniceanu

50 REVISTA DE MATEMATICĂ MEHEDINȚEANĂ NR. 15


S.S.M.ROMÂNIA - Filiala Mehedinți 2015
VIII.29. Să se rezolve ecuația: ∆ − (∆ + ) + = știind că − + =

Prof. Dana Paponiu

VIII.30. Să se afle știind că: − = . Prof. Marica Ștefan

VIII.31. Să se găsească ≥ astfel încât + = .

Prof. Carmen Chirfot


ʹ
VIII.32. În tetraedrul , aria și perimetrul bazei sunt √ , respectiv . Fie ⊥
ʹ ʹ
; ⊥ ; ⊥ ; ∈ ( ); ʹ ∈ ( ); ʹ ∈ (
ʹ ) și ʹ
= ʹ
= ʹ
= √ . Se cere
volumul tetraedrului. Prof. Daniel Sitaru

VIII.33. Fie > 0. Să se arate că: + + > + . Prof. Daniel Sitaru


PA +PB+PC
VIII.34. Demonstrați că: a) PC +PB < AB + AC , ∀ ∈ (∆ ) b) q ∙p
< VA + VB + VC

∀ ∈ ( ), tetraedru , ∈ ℕ∗ , p și q sunt numere naturale prime.

Prof. Dana Paponiu

Clasa a IX-a
IX.1. Fie , , ∈ ℝ∗ , cu proprietatea = . Să se arate că ( + ) ≥ √ − . Să se
precizeze cazul în care avem egalitate. Prof.dr. Gheorghe Căiniceanu

IX.2. Fie numerele reale , , , ∈ [− , ] astfel încât + + + = . Să se demonstreze că:


+ + + ≥− . Prof. Diana Trăilescu(Danubius Contest)

IX.3. Fie numerele reale , , , ∈ [− , ] astfel încât + + + = .

Să se determine + + + . Prof. Diana Trăilescu

IX.4. Să se arate că dacă ≥ , atunci: + ≥ .

Prof. Daniel Sitaru


IX.5. Fie , , , măsurile, în radiani, ale patrulaterului convex . Să se arate că:

+ + + + + + + + + + + ≥ √

Prof. Claudia Nănuți


IX.6. Să se determine funcția : ℝ ⟶ ℝ cu proprietatea: + ≤ ≤ ( )+ .
Prof. Claudia Nănuți
IX.7. Să se determine ∈[ , ∞) astfel încât are loc relația:

√ − + + √ − + + ⋯+ √ − +
=

51 REVISTA DE MATEMATICĂ MEHEDINȚEANĂ NR. 15


S.S.M.ROMÂNIA - Filiala Mehedinți 2015
unde [ ] reprezintă partea întreagă a numărului real . Prof. Ovidiu Ticuși

IX.8. Fie ∆ ascuțitunghic înscris în cercul de centru și de rază = cm, iar ʹ punctul
diametral opus lui . Dacă este ortocentrul ∆ , iar este ortocentrul ∆ ʹ , arătați că
ʹ

⃗ = ʹ ⃗ʹ și aflați modulul vectorului ⃗ʹ . Prof. Ovidiu Ticuși

IX.9. Calculați termenul general al șirului ( ) cu = și − = ( )

Prof. Gabriela Bondoc

IX.10. Să se arate că în orice triunghi este adevărată relația:


(| |−| |) + (| |−| |) + (| |−| |) ≤

Prof. Daniel Sitaru


IX.11. i) Demonstrați că ∀ , , ∈ are loc inegalitatea ( + + ) ≥ ( + + )

ii) Fie numerele strict pozitive , , astfel încât + + = . Să se demonstreze


inegalitatea: + + ≥ ( + + ) Prof. Leonard Giugiuc

IX.12. Să se arate că pentru orice numere reale , , , și are loc inegalitatea:

+ + + + + + −( + ) +( + ) ≥ + +

Când are loc egalitatea? Prof. Leonard Giugiuc

IX.13. În triunghiul de laturi , , lungimile bisectoarelor interioare sunt , , , . Să se arate că


dacă: = = atunci triunghiul este echilateral. Prof. Claudia Nănuți

ʹ ʹ
IX.14. Fie bisectoare în triunghiul .Să se arate că: < Prof. Daniel Sitaru
ʹ
IX.15. Să se arate că dacă ecuațiile + + = și + + ʹ = au rădăcinile , ,
ʹ ʹ ʹ ʹ
respectiv , atunci ecuația: + + + + = are rădăcinile

ʹʹ ʹ ʹʹ ʹ ʹ ʹ ʹ
= + ; = + dacă și numai dacă + = . Prof. Ion Nănuți

IX.16. Să se arate că dacă: , , , , , ∈ ℚ și

+ √ + √ = + √ + √ atunci = ; = ; = . Prof. Claudia Nănuți

IX.17. Să se arate că în orice triunghi de laturi , , avem: + + ≥ − .

Prof. Daniel Sitaru

IX.18. Să se arate că dacă , , ∈ ( , ∞) atunci: + + ≥ √ .

Prof. Daniel Sitaru

IX.19. Să se rezolve ecuația: + [ ] − [ ]{ } − { } = , unde [ ], { } sunt partea


întreagă, respectiv partea fracționară a numărului real . Prof. Daniel Sitaru

52 REVISTA DE MATEMATICĂ MEHEDINȚEANĂ NR. 15


S.S.M.ROMÂNIA - Filiala Mehedinți 2015
IX.20. Să se arate că în orice triunghi avem relația:

+ + ≥ + + Prof. Daniel Sitaru


√ √ √

IX.21. Să se demonstreze că: ≤ + +⋯+ < ; ∈ ℕ∗ . Prof.Dan Nănuți

IX.22. Să se rezolve ecuația: { }= +[ ] Prof.Dan Nănuți

IX.23. Să se rezolve ecuația: [ + ]+ { }= Prof.Dan Nănuți

IX.24. În ce fel de triunghi , laturile , , sunt direct proporționale cu , , ?


Dar cu , , ? Prof. Daniel Sitaru

IX.25. Se consideră vectori în plan cu proprietatea că oricare dintre ei este un vector coliniar cu
suma celorlalți . Să se arate că toți vectorii sunt coliniari. Prof. Claudia Nănuți
IX.26. Să se arate că în orice triunghi avem relația: ( − )( − )( − ) ≤

Prof. Claudia Nănuți



IX.27. Să se arate că în orice triunghi avem: ( + + )≤ + +

Prof. Daniel Sitaru

IX.28. Să se arate că în orice triunghi avem: ≤ . Prof. Daniel Sitaru

IX.29. Să se arate că în orice triunghi: + ≥ + . Prof. Daniel Sitaru

IX.30. Fie ∈ ℕ∗ . Rezolvați în ℕ ℕ ecuația:

( + ) ( ) ( )
( + )+ + +⋯+ ( + )= ( + )

Prof. Daniel Sitaru


IX.31. În ∆ având lungimile laturilor numere naturale consecutive, ⊥ ; , − razele
cercurilor circumscrise ∆ , respectiv ∆ . Dacă + = și = să se arate că
= . Prof. Dana Paponiu

IX.32. Fie un triunghi isoscel cu = . Pe baza se ia ∈( ); = și notăm


= . Să se exprime în funcție de . Dacă = să se arate că < .

Prof. Marica Ștefan

IX.33. Să se rezolve ecuația: − − + = , unde , , , sunt termeni


consecutivi ai unei progresii geometrice de numere reale nenule și + = , iar

( ) = . Prof. Dana Paponiu

IX.34. Să se demonstreze că (∀) , , ∈ ( , ∞) avem: ( + )( + )( + )( + + ) ≥

Prof. Dorel Căpraru

53 REVISTA DE MATEMATICĂ MEHEDINȚEANĂ NR. 15


S.S.M.ROMÂNIA - Filiala Mehedinți 2015
IX.35. Fie , , , ∈ ( , ∞) și ∈ ℕ∗ .

Adevărat sau fals:


( + + )≤ + + +

Prof. Daniel Sitaru


IX.36. Să se arate că dacă , , , ∈ ( , ∞) atunci


+ +

Prof. Dan Nănuți


Clasa a X-a

X.1. Dacă > > 0 și + ≤ √ ( − ) arătați că: + ( + )≥ ( + )

Prof. Leonard Giugiuc (Olimpiada Mongolia -2015)

X.2.. La o conferință participă persoane. Să se arate că există două persoane care au exact
același număr de prieteni printre participanți. Prof.dr. Gheorghe Căiniceanu

X.3. Fie numerele reale fixate , > 0. Să se arate că: ∀ , , ∈ [ , ] are loc inegalitatea:
|( − ) +( − ) | + |− + | + |( + − ) − |≥
+

Prof. Leonard Giugiuc(Danubius Contest)

X.4. Să se determine centrul radical al cercurilor de ecuații:

: +( + ) +( − ) − = ; : +( − ) +( + ) + =

: +( + ) +( − ) − = . Prof. Daniel Sitaru

X.5. Fie ∈ ℂ astfel încât | | ≤ . Să se arate că:

+√ ∙ + ∙ + √ ∙ +⋯+ √ ∙ < Prof. Ovidiu Ticuși

X.6. Să se arate că: ∏ !∙∏ = ( !) Prof. Ion Nănuți

X.7. Să se arate că ecuația: ! + !=( + )! nu are soluție. Prof. Daniel Sitaru

X.8. Să se demonstreze că: ( − )>3 !. Prof. Daniel Sitaru



X.9. Fie ∈ ℂ∗ încât + = + ; = + . Calculați: + ; ∈ ℕ∗ .Prof. Daniel Sitaru

X.10. Să se arate că dacă , , ∈ ( , ∞), atunci: < ( + + )

Prof. Daniel Sitaru

54 REVISTA DE MATEMATICĂ MEHEDINȚEANĂ NR. 15


S.S.M.ROMÂNIA - Filiala Mehedinți 2015
X.11. Să se arate că în orice triunghi avem relațiile: + + ≥ + +

și + + ≥ . Prof. Claudia Nănuți

X.12. Numărul diagonalelor unui poligon convex este egal cu numărul punctelor de intersecție a
diagonalelor situat în interiorul poligonului. Câte laturi are poligonul? Prof. Daniel Sitaru

X.13. Rezolvați ecuațiile: ( + )( + )( + ) = ; √ + −√ − = ;

∙ = Prof. Ecaterina Pupăză


X.14. Să se calculeze: | − | + | − | + − , unde este o rădăcină a ecuației
+ + = . Prof. Ecaterina Pupăză

X.15. Să se reprezinte grafic mulțimea punctelor din plan ale căror afixe verifică simultan
relațiile: ≤ ≤ și ≤ | − − | ≤ . Prof. Ecaterina Pupăză

X.16. Să se rezolve ecuația: + + = . Prof. Daniel Sitaru

X.17. Fie , , măsurile, în radiani, ale unghiurilor unui triunghi. Să se arate că:
+ + + +
≤ ; ≤
+ + + +

Prof. Daniel Sitaru


!
X.18. Să se arate că fracția: este subunitară. Prof. Ovidiu Ticuși

X.19. Să se arate că: < ! Prof.Dan Nănuți

X.20. Poligoanele și sunt regulate și au aceeași orientare. Demonstrați că măsura


unghiului ascuțit dintre și este . Prof.Ruben Dario

X.21. Fie un triunghi și ∈( ). și centrele cercurilor înscrise în ∆ respectiv ∆ .


Dacă este inscriptibil, demonstrați că: = . Prof. Miguel Ochoa Sanchez

X.22. Fie , ,…, ∈ ℂ. Să se arate că: ≤| + |+| + | + ⋯+ | + |+| |

Prof. Claudia Nănuți

X.23. Considerăm mulțimea = − + : , ∈ ℤ . Demonstrați că dacă ∈ atunci


∈ . Prof. Leonard Giugiuc

X.24. Fie un triunghi echilateral și cercul înscris în triunghi. Considerăm că ∈ și notăm


cu , , distanțele de la la , , respectiv . Demonstrați că: = .

Prof. Miguel Ochoa Sanchez

X.25. Fie , , > 0 numere reale. Demonstrați că:

55 REVISTA DE MATEMATICĂ MEHEDINȚEANĂ NR. 15


S.S.M.ROMÂNIA - Filiala Mehedinți 2015
∙ + + − − −
+ + ≥ + + +
+ +

Prof. Leonard Giugiuc

X.26. Fie ∈ ℝ, -fixat, ∈ , . Fie ∈ ( ) , − , încât: (∡( )) =


și (∡( )) = − . Găsiți toate valorile posibile ale (∡( )).

Prof. Leonard Giugiuc

X.27. Dacă , , ≥ , demonstrați că:

( + )( + )( + ) ( + ) ≥( + + )( + + )+

Prof. Leonard Giugiuc

X.28. Fie , , și un număr fix real astfel încât + , , , > 1, ≥ și > . Demonstrați
că ecuația + = [ + ( ) ] are cel mult o soluție reală.

Prof. Leonard Giugiuc(Danubius Contest)

X.29. Să se rezolve ecuația:

+ +
=
( + )( + )!

Prof. Claudia Nănuți

X.30. Să se calculeze restul împărțirii numărului la numărul .


Prof. Piț-Rada Ionel-Vasile
X.31. Să se arate că (∀) ∈ ℕ∗ ; ≥ avem: √ !+ √ !+⋯+ √ ! ≤ + −

Prof. Daniel Sitaru

X.32. Să se demonstreze că (∀) ∈ ℕ∗ :


+
+ + + + ⋯+ + <1
‼ ‼ ‼ ‼ ( + )‼ ( + )‼

Prof. Daniel Sitaru

X.33. Să se determine ∈ ℕ∗ astfel încât:

− =
√ ! + +√ + √

Prof. Daniel Sitaru

56 REVISTA DE MATEMATICĂ MEHEDINȚEANĂ NR. 15


S.S.M.ROMÂNIA - Filiala Mehedinți 2015

X.34. Fie ( ) = + √ + √ + + − √ . Aflați ∈ ( , ∞) încât rădăcinile


polinomului să aparțină discului de rază . Prof. Emilia Răducan

X.35. Fie , ∈ ℝ și , ∈ ℂ. Să se arate că:

( + ) | | +( + ) | | − | + | =| − |

Prof. Claudia Nănuți

X.36. Fie , ∈ ℂ. Să se arate că:

| + | | | | |
≤ +

Prof. Claudia Nănuți

Clasa a XI-a

XI.1. Fie : ℝ → ℝ, continuă iar ( )= , →∞ →∞ ( ) = ( , ∈ ℝ). a) Arătați că este



marginită pe ℝ. b) Să se arate că ∀ ∈ ℕ ∃ ∈ ℝ, a.î. ( ) = + + . c) Arătați că șirul
( ) conține un subșir convergent. (Șirul ( ) este șirul obținut la b).

Prof. dr. Gheorghe Căiniceanu

XI.2. Să se arate că șirul: , ∈ ℝ∗ , = este mărginit. Prof. Daniel Sitaru

XI.3. Să se studieze convergența sirului:


!
= ; ≥
( + )( + )…( + )

În cazul în care șirul este convergent, să se determine limita sa. Prof. Daniel Sitaru

XI.4. Să se calculeze:

→ + ( + )
Prof. Daniel Sitaru
XI.5. Să se calculeze:
√ +√
→ √ +√
Prof. Claudia Nănuți
XI.6. Să se calculeze:
∙ !
;
→ ∙ ! →

Prof. Claudia Nănuți

57 REVISTA DE MATEMATICĂ MEHEDINȚEANĂ NR. 15


S.S.M.ROMÂNIA - Filiala Mehedinți 2015

XI.7. Să se demonstreze că: < <

Prof. Daniel Sitaru


XI.8. Să se determine valorile lui ∈ ℕ∗ astfel încât ( + ) = .

Prof. Daniel Sitaru

XI.9. Fie : ℝ → ℝ; ( ) = + − . Să se arate că nu există

( )ș ( )
→∞ → ∞

Prof. Daniel Sitaru


XI.10. Fie : ℝ → ℝ o funcție neconstantă al cărei grafic admite două axe de simetrie paralele cu
axa . Să se arate că nu există:

( ); ( ).
→∞ → ∞

Prof. Claudia Nănuți

XI.11. Să se studieze convergența șirului:

= ; = ( + )( − )

Prof. Daniel Sitaru

XI.12. Să se calculeze:
( − ) −( − )
→ ( − ) −( − )

Prof. Ecaterina Pupăză


XI.13. Să se calculeze:
+ + + ⋯+ −
;
→ →

Prof. Ecaterina Pupăză


XI.14. Să se arate că în orice triunghi ascuțitunghic :
+ + > și + + < . Prof.Dan Nănuți

XI.15. Se cere cilindrul de volum maxim care se poate înscrie în conul de rază 5 și înălțime 12.
Prof. Claudia Nănuți
(ℂ); ∗ ( − )= .
XI.16. Fie ∈ = = ; = . Să se rezolve ecuația:

Prof. Daniel Sitaru


XI.17. Fie , ∈ (ℂ). Să se arate că:

− + + + = ( + + ( + ))

58 REVISTA DE MATEMATICĂ MEHEDINȚEANĂ NR. 15


S.S.M.ROMÂNIA - Filiala Mehedinți 2015
Prof. Claudia Nănuți
XI.18. Fie , ∈ (ℝ); ( + )≠ și + = √ − ( − ) . Arătați că este
multiplu de . Prof. Sitaru Daniel(Danubius Contest)

XI.19. Fie , ∈ (ℂ) matrici nesingulare astfel încât există ∈ ℕ∗ cu proprietatea


= . Să se arate că = . Prof. Daniel Sitaru

XI.20. Fie ( ,∙) grupul permutărilor de ordin . Să se afle știind că numărul tuturor inversiunilor
permutărilor din este . Prof. Daniel Sitaru

XI.21. Se consideră sistemul:


( − )+ ( − ) =
( + ) + ( + ) =

Să se determine: a.Valorile ∈ ℤ încât ∈ ℤ sau ∈ ℤ, b.Valorile ∈ ℤ încât ∈ ℤ și ∈ℤ

c.Valorile ∈ ℤ încât ∈ℤ Prof. Ion Nănuți

XI.22. Fie = ; ∈ ℝ. Să se arate că nu există ∈ ℤ încât elementele lui să fie toate


numere întregi. Prof. Dorel Căpraru

XI.23. Fie ∈ (ℂ). Să se arate că:

( − )= ∗
+ − +

( − )= ( ∗)
+ − +

Prof. Claudia Nănuți


XI.24. Să se calculeze:
− − − ( − )
; ∈ ℕ;
→ − → ( − )

Prof. Ecaterina Pupăză


XI.25. Să se afle numerele reale și astfel ca dreapta = + să fie asimptotă spre ∞ pentru
funcția : → ℝ, ( ) = ( este domeniul maxim de definiție);
Prof. Ecaterina Pupăză
XI.26. Să se calculeze:

+
→∞

Prof. Dan Nănuți


XI.27. Fie ( ) un șir de numere reale pozitive nenule, astfel încât:

= .
→∞

Să se calculeze:

59 REVISTA DE MATEMATICĂ MEHEDINȚEANĂ NR. 15


S.S.M.ROMÂNIA - Filiala Mehedinți 2015

.
→∞

Prof. Daniel Sitaru

(ℝ) cu ( ∗)
XI.28. Fie ∈ = = .Să se arate că: + + ≥

Prof. Daniel Sitaru


XI.29. Să se arate că există ∈ ℕ încât + + ⋯+ ≥ , unde [ ] – reprezintă
partea întreagă a numărului real . Prof. Daniel Sitaru
XI.30. Să se găsească ∈ (ℝ); ≠ încât = . Prof. Daniel Sitaru

XI.31. Fie = + + ∙…∙ + ; ∈ ℕ∗. Să se arate că șirul ( ) este


convergent. Prof. Daniel Sitaru

XI.32. Să se calculeze:


→∞ ( − )

Prof. Daniel Sitaru

XI.33. Fie , ∈ (ℝ), astfel încât = . Demonstrați inegalitatea:

( − + )+ ( + + )≥| + | Prof. Leonard Giugiuc

XI.34. Rezolvați în (ℝ) și în (ℤ) ecuația: = . Prof. Leonard Giugiuc

XI.35. Fie , , numere reale astfel încât + + = . Demonstrați că:

≥ − − − Prof. Leonard Giugiuc

XI.36. Demonstrați că ∀ ∈ [ , ] și ∀ , , > are loc inegalitatea:

+ ( + )( + )( + )
≥ ∙ +( − )√

Prof. Leonard Giugiuc

XI.37. Fie ≥ un număr natural și > , numere reale, ≤ ≤ , astfel încât: ∏ =

Demonstrați că: ∑ ≤ , pentru = , . Prof. Leonard Giugiuc


XI.38. Dacă ≥ și ∈ ( , ), atunci demonstrați că: ∑ ≤


Prof. Dorin Mărghidanu

XI.39. Fie numerele reale , cu < < și șirul real astfel încât:

60 REVISTA DE MATEMATICĂ MEHEDINȚEANĂ NR. 15


S.S.M.ROMÂNIA - Filiala Mehedinți 2015

+ = √
→∞

Studiați existența:

→∞

Prof. Leonard Giugiuc


XI.40. Să se calculeze:

Prof. Dan Nănuți


XI.41. Să se calculeze limita șirului:

+ +
=
+

Prof. Claudia Nănuți


XI.42. Să se calculeze limita șirului:

+ +
=
( + )( + )!

Prof. Claudia Nănuți

XI.43. Fie = ∈ ; ( ) ≠ ; ∈ , ; ∈ ℕ; ≥ . Calculați: | |+| |+| |.

Prof. Emilia Răducan

XI.44. Fie numărul maxim de inversiuni al unei permutări ∈ ; ∈ ℕ; ≥ și , , ∈ ℝ,

+ + = . Să se calculeze:

( + + ).
→∞

Prof. Daniel Sitaru


XI.45. Fie mulțimea tuturor matricelor pătratice de ordin , ale căror elemente sunt numerele
reale , , … , , fiecare exact o dată. a. Determinați ( ); b. Calculați ∑ ∈ .

Prof. dr. Gheorghe Căiniceanu

XI.46. Să se arate că (∀) ∈ ℕ; ≥ avem: ( + ) + ∙ …∙ ( + )≥ +√

Prof. Daniel Sitaru



XI.47. Să se arate că dacă + ≥ atunci: + ≥ ; ∈ℕ. Prof. Daniel Sitaru

XI.48. Să se arate că dacă + ≥ și + ≥ atunci + + + ≥ ,

61 REVISTA DE MATEMATICĂ MEHEDINȚEANĂ NR. 15


S.S.M.ROMÂNIA - Filiala Mehedinți 2015
(∀) , , , ∈ ℝ. Prof. Daniel Sitaru

Clasa a XII-a

XII.1. Fie : ℝ ⟶ ℝ, ( ) = | − |. Să se calculeze:


∈ℤ

( )
⟶∞

Prof. Daniel Sitaru



XII.2. Calculați integrala definită: = ∫ . Prof. Ovidiu Ticuși

XII.3. Calculați:

∙ ∙

Prof. Ovidiu Ticuși


XII.4. Să se calculeze: =∫ ( ) ( )
;

∈ ; ⊆ ℝ∖{ | ( + ) ( + )≠ } Prof. Claudia Nănuți


XII.5. Să se calculeze:

( )
→∞

Prof. Daniel Sitaru


( )
XII.6. Să se calculeze: =∫ ; ∈ ; ⊆ ℝ∖{ | ∈ ℤ}.

Prof. Daniel Sitaru


XII.7. Să se determine numărul structurilor de grup ce se pot defini pe o mulțime cu elemente.

Prof. Dan Nănuți


XII.8. Fie = | ∈ ℂ; = . Să se arate că ( ,∙) ≃ (ℤ , +).

Prof. Dan Nănuți


XII.9. Fie ( ,∙) grup și , ∈ . Știind că = și = să se arate că = .
Prof. Mihai Octavian Ungureanu
XII.10. Fie = | ∈ (ℝ); ʹʹ ( ) − ʹ ( ) + ( ) = , (∀) ∈ ℝ . Definim pe adunarea
funcțiilor astfel: ( + )( ) = ( ) + ( ). Să se arate că ( , +) este grup abelian.

Prof. Dan Nănuți


XII.11. Pe mulțimile = ( , ∞) și = ( , ∞) definim legile de compoziție:

∗ = − − + respectiv ∘ = − − + .

Să se arate că ( ,∗) și ( ,∘) sunt grupuri abeliene izomorfe. Prof. Dan Nănuți

62 REVISTA DE MATEMATICĂ MEHEDINȚEANĂ NR. 15


S.S.M.ROMÂNIA - Filiala Mehedinți 2015
XII.12. Se consideră familia de drepte:

:( − ) + ( + ) + − = ; ∈ℝ

Fie ={ | ∈ ℝ}. Definiți pe o lege de compoziție internă „ ∗ ” astfel încât ( ,∗) ≃ (ℝ, +).

Prof. Dorel Căpraru

+ =
XII.13. Să se rezolve în ℤ ℤ sistemul: Prof. Dorel Căpraru
+ =

XII.14. Fie = ∈ ℝ∗ și = ∈ ℝ∗ .Studiați dacă ( ,∙) ≃ ( ,∙)

Prof. Dorel Căpraru

XII.15. Pe mulțimea ℝ definim legea de compoziție:

∗ = √ + −√

Să se arate că(ℝ,∗) este grup abelian și (ℝ,∗) ≃ (ℝ, +). Prof. Claudia Nănuți

XII.16. În grupul ( ,∙) avem relațiile: ( ) =( ) și ( ) =( ) ; (∀) , ∈ .


Să se arate că ( ,∙) este grup abelian. Prof. Claudia Nănuți

XII.17. Fie = { , , − , − }; = − . Să se arate că grupul ( ,∙) nu este izomorf cu grupul lui


Klein ( ,∘). Prof. Mihai Octavian Ungureanu

XII.18. Fie = ( , ∞). Definim pe legile de compoziție: ∗ = și ∘ = √ . Să se arate


că grupurile ( ,∗) și ( ∖ { },∘) nu sunt izomorfe. Prof. Claudia Nănuți

XII.19. Fie ={ ∈ | ( ) = }. Să se arate că ( ,∘) nu admite subgrupuri de ordin 18.

Prof. Claudia Nănuți

XII.20. Fie : ℝ → ℝ; ( ) = { } − { } și ={ | > 0; ( + ) = ( ); ∈ ℤ}.

Să se arate că admite primitive pe ℝ și ( , +) este grup abelian. Prof. Daniel Sitaru

XII.21. Fie ={ :( , ∞) → ℝ| ( )= +( − ) ; ∈ ℝ∗ }.

Să se arate că ( ,∘) este grup abelian și ( ,∘) ≃ (ℝ∗ ,∙).

Prof. Mihai Octavian Ungureanu


XII.22. Pe = ( , ∞) definim legea de compoziție:

∘ =

Să se arate că ( ,∘) este grup abelian și ( ,∘) ≃ (ℝ, +). Prof. Daniel Sitaru

63 REVISTA DE MATEMATICĂ MEHEDINȚEANĂ NR. 15


S.S.M.ROMÂNIA - Filiala Mehedinți 2015

XII.23. Fie = | : ℝ → ℝ; + ( ) = + + ( ) ; , ∈ ℝ . Să se arate că ( ,∘) este


grup abelian. Prof. Daniel Sitaru

XII.24. Să se calculeze:

∫ [ ]
=
→∞

unde [ ] − este partea întreagă a numărului real . Prof. Daniel Sitaru

XII.25. Să se demonstreze că:

( ) >

Prof. Daniel Sitaru


XII.26. Considerăm polinomul ∈ ℝ[ ], = + + +( − ) .

a. Determinați tripletele de numere reale ( ; ; ), cu ≠ , pentru care


b. = + + +( + ) . b. Să se afle valoarea minimă a funcției
: ℝ → ℝ, ( ) = + + ( ), ∀ ∈ ℝ, unde este funcția polinomială asociată lui .

Prof. Leonard Giugiuc(Danubius Contest)

CONCURSUL NAȚIONAL DE MATEMATICĂ APLICATĂ „ADOLF HAIMOVICI”


ETAPA LOCALĂ-14.03.2015
Clasa a IX-a PROFIL SERVICII
SUBIECT I: Fie , , , ∈ ( , ∞). Să se arate că:

+ + + + + + + ≥

Prof. Daniel Sitaru-RMM-14/2014


SUBIECT II : Fie = + + ⋯+ ; ∈ ℕ∗.Să se arate că: + + ⋯+ divide

Prof.Daniel Sitaru-RMM-14/2014
=
SUBIECT III: Să se rezolve sistemul: − = . Prof. Daniel Sitaru- GM-3/2015
− =

SUBIECT IV: Să se determine funcțiile : ℝ → ℝ cu proprietatea:


( )+ ( )= ( ); ∀ , ∈ ℝ

Prof.Daniel Sitaru-RMM-14/2014

64 REVISTA DE MATEMATICĂ MEHEDINȚEANĂ NR. 15


S.S.M.ROMÂNIA - Filiala Mehedinți 2015

Clasa a X-a PROFIL SERVICII


SUBIECT I: Fie : ℝ → ℝ , astfel încât : ( ∘ ∘ )( ) − ( ∘ )( ) + ( )− = ,∀ ∈ ℝ

Să se arate că funcția este injectivă. Prof.Daniel Sitaru-RMM-14/2014

SUBIECT II: Să se determine numerele reale strict pozitive, , , … , , …, care verifică relația:
+ + ⋯+ =( + )( + +⋯+ ), pentru orice ∈ ℕ∗ .

Prof.Daniel Sitaru-GM-3/2014

SUBIECT III: Fie ∈ ℕ∗ și ∈ , ; ∈ , . Să se arate că:

+ ≤

Prof.Daniel Sitaru-RMM-14/2014
SUBIECT IV: Să se rezolve ecuațiile: − ∙ + = ;

+ + + = ; + + + + =

Prof.Claudia Nănuți

Clasa a XI-a PROFIL SERVICII


SUBIECT I: Fie , , , ∈ ( ); + = + ; + = + ;

= și = . Să se arate că: + = + ; ∀ ∈

Prof.Claudia Nănuți-RMM-14/2014
SUBIECT II: Să se calculeze limita șirului:

= −
− √ − + √ −

Prof.Claudia Nănuți-RMM-14/2014
SUBIECT III: Fie , , ∈ ( ); ≠ și = . Să se determine ∈ ( )

încât = . Prof.Daniel Sitaru-RMM-14/2014

SUBIECT IV: Fie : [ , ] → ℝ o funcție continuă astfel încât ( ) = ( ). Arătați că există


∈ ( , ) astfel încât ( ) = ( + ). Prof.Daniel Sitaru-GM-3/2014

Clasa a XII-a PROFIL SERVICII

SUBIECT I: Fie = ℝ/ − ; ∗ = + + . Să se arate că ( ,∗) este grup abelian



și (ℝ ,∙) ≅ ( ,∗). Prof.Claudia Nănuți-RMM-14/2014

65 REVISTA DE MATEMATICĂ MEHEDINȚEANĂ NR. 15


S.S.M.ROMÂNIA - Filiala Mehedinți 2015
SUBIECT II: Să se demonstreze că:

∙ ≥

Prof.Daniel Sitaru-RMM-14/2014
SUBIECT III: Pe mulțimea = ( , ∞) definim legea de compoziție: ∗ = − − +

Să se rezolve ecuațiile: ∗ = ; ∗ ∗ = ; ∗ ∗ ∗ = si = ′ unde ′ este simetricul


elementului față de legea " ∗ " . Prof.Claudia Nănuți

SUBIECT IV: Să se calculeze: =∫ . Prof.Daniel Sitaru

Clasa a IX-a PROFIL ȘTIINȚE ALE NATURII


ʹ ʹ
SUBIECT I: Fie , ∈ ( , ∞). Pe laturile ∆ se consideră , , ʹ astfel încât:

ʹ⃗= ∙ ⃗ʹ ; ʹ ⃗ = ∙ ⃗ ; ʹ ⃗ = ∙ ⃗ʹ
+
ʹ ʹ
Să se arate că , , ʹ sunt concurente. Prof.Daniel Sitaru

SUBIECT II: Fie = [ , ]; = { , }. Să se reprezinte grafic produsele carteziene:


, , , . Prof.Claudia Nănuți

SUBIECT III: Să se rezolve ecuațiile:

a.[| + | − ] = , b.|[ + ] − | =

unde [ ]-reprezintă partea întreagă a numărului real . Prof.Dan Nănuți


SUBIECT IV: a. Într-o progresie aritmetică = . Se cere suma primilor trei termeni.

b.Într-o progresie geometrică = . Se cere produsul primilor trei termeni.


Prof.Emilia Răducan

Clasa a X-a PROFIL ȘTIINȚE ALE NATURII


SUBIECT I:Să se determine ∈ [ , ∞) astfel încât: + √ + − √ =

Prof. Mihai Octavian Ungureanu

SUBIECT II: Să se arate că: ∙ ∙ = . Prof. Dan Nănuți


SUBIECT III: Fie , , ∈ ℂ ; | | = | | = | | ≠ . Să se arate că:

( + )( + )( + )
∈ℝ

Prof.Giugiuc Leonard

66 REVISTA DE MATEMATICĂ MEHEDINȚEANĂ NR. 15


S.S.M.ROMÂNIA - Filiala Mehedinți 2015
SUBIECT IV: Să se reprezinte grafic mulțimea punctelor din plan ale caror afixe verifică simultan
relațiile: | − | ≤ , | + | ≤ . Prof. Emilia Răducan

Clasa a XI-a PROFIL ȘTIINȚE ALE NATURII


SUBIECT I: Fie = si = { ( )| ( ) = + ; ∈ ℝ}.

a. Să se arate că: ( ) ( ) = ( + + ), ∀ , ∈ℝ
b. Să se afle ∈ ℝ încât:

− ∙ − ∙ …∙ ∙ ∙ = ( ) Prof. Mihai Octavian Ungureanu

SUBIECT II : Fie

=
!

Să se arate că:

( + )! ( − )=
→∞

Prof.Daniel Sitaru

SUBIECT III: Fie = ; = .

a.Să se calculeze , ,…, ; ≥ .Să se calculeze , ,…, ; ≥ Prof. Dan Nănuți


SUBIECT IV: Să se calculeze:

( ) − +
; ; ;
→ → ( ) →∞ + →∞ −

Prof. Emilia Răducan

Clasa a XII-a PROFIL ȘTIINȚE ALE NATURII


SUBIECT I: Fie = si = { ( )| ( ) = + ; ∈ ℝ}.

c. Să se arate că: =
d. Să se arate că: ( ) ( ) = ( + + ), ∀ , ∈ℝ
e. Să se arate că: ∀ ∈ ℤ, ( ) este inversabilă.

Prof. Mihai Octavian Ungureanu


SUBIECT II: Fie , : ℝ → ℝ ; , continue pe ℝ și , ∈ ℝ . Știind că:

∀ ∈ ℝ , ( ) = ( − ) și ( )+ ( )= să se calculeze:

( )+ ( )
=
( )+ ( )+

67 REVISTA DE MATEMATICĂ MEHEDINȚEANĂ NR. 15


S.S.M.ROMÂNIA - Filiala Mehedinți 2015
Prof.Daniel Sitaru-RMM-14/2014
SUBIECT III: Fie = ( , ∞). Definim pe legea de compoziție: ∗ = − − +

a. Să se afle , ∈ ℚ/ℤ încât ∗ ∈ ℤ


b. Să se afle , ∈ ℝ/ℚ încât ∗ ∈ ℚ Prof.Dan Nănuți
SUBIECT IV: Fie =∫ ; ∈ ( , ∞).

a. Calculați: ,
b. Determinați o relație de recurență pentru calculul lui . Prof.Giugiuc Leonard

Clasa a IX-a PROFIL TEHNIC


SUBIECT I: Să se rezolve ecuațiile: . | − | = . | − | + | − | = . | − | = −

Prof.Dan Nănuți

SUBIECT II: Să se rezolve ecuațiile: . [ ] = , . = , . = .

Prof.Giugiuc Leonard

SUBIECT III: Fie ∆ , mijlocul segmentului [ ], mijlocul segmentului [ ], mijlocul


segmentului [ ]. a.Să se arate că ⃗ = ⃗ + ⃗ . b.Să se arate că ⃗ + ⃗ + ⃗ = ⃗.

Prof. Doru Preșneanu

SUBIECT IV: Fie un hexagon regulat de centru și un punct în planul hexagonului.


Să se arate că: ⃗+ ⃗+ ⃗+ ⃗+ ⃗+ ⃗ = ⃗. Prof.Daniel Sitaru

Clasa a X-a PROFIL TEHNIC


SUBIECT I: Fie , ∈ [ , ∞). Să se arate că ∀ , ∈ ℝ : ∙ ≤

Prof. Daniel Sitaru-RMM-14/2014

SUBIECT II: Să se rezolve ecuațiile: . = , . + = , . − − =

Prof.Dan Nănuți
SUBIECT III: Să se rezolve ecuația: + = .

Prof.Daniel Sitaru-RMM-14/2014

SUBIECT IV: Să se rezolve ecuațiile: . √ − = , . − + = , . √ − +√ − =

Prof. Mihai Octavian Ungureanu

Clasa a XI-a PROFIL TEHNIC


SUBIECT I: Fie , , , ∈ (ℂ); + = + ; + = + ; = și = .
Să se arate că: + = + ; ∀ ∈ ℕ∗ . Prof.Claudia Nănuți-RMM-14/2014

68 REVISTA DE MATEMATICĂ MEHEDINȚEANĂ NR. 15


S.S.M.ROMÂNIA - Filiala Mehedinți 2015

SUBIECT II: Fie = ; = . Să se arate că:

.( + ) = + , . ( )= ∙ . Prof.Dorel Căpraru

SUBIECT III: Fie , ∈ ℝ și funcția : ℝ → ℝ, ( ) = + + + . Determinați și , astfel


încât:

( )=
→∞

Prof. Dan Nănuți

SUBIECT IV: Să se calculeze:


− + +
. , . , .
→ + → − → −

Prof. Emilia Răducan

Clasa a XII-a PROFIL TEHNIC


SUBIECT I: Să se calculeze:

√ − + +
, ∈ −√ , √
√ −
Prof.Giugiuc Leonard

SUBIECT II: Fie = ( ) ( )= ; ∈ ℝ . Să se arate că ( ,∙) este grup abelian și


( ,∙) ≅ (ℝ , +) . Prof.Daniel Sitaru-RMM-14/2014

SUBIECT III: Pe mulțimea = ( , ∞) definim legea de compoziție: ∗ = − − +

Să se rezolve ecuațiile: ∗ = ; ∗ ∗ = ; ∗ ∗ ∗ = și = ′ unde ′ este simetricul


elementului față de legea " ∗ " . Prof.Claudia Nănuți

SUBIECT IV: Fie =∫ ; + ≠ ; ∀ ∈ [ , ]; ∈ ℕ∗ . Să se calculeze:

( ∙ )

Prof.Claudia Nănuți-RMM-14/2014

Clasa a IX-a PROFIL UMAN


SUBIECTUL I: a. Să se arate că ( + ) + ( − ) ≥ + , ∀ , ∈ ℝ
b. Dacă ∈ [− , ] și ∈ [ , ], să se arate că + ∈ [ , ].
Prof. Doru Preșneanu

SUBIECTUL II: a. Să se rezolve în ℤ ecuația − =− .


b.Să se rezolve în ℕ ecuația | + | − = . Prof. Doru Preșneanu

69 REVISTA DE MATEMATICĂ MEHEDINȚEANĂ NR. 15


S.S.M.ROMÂNIA - Filiala Mehedinți 2015
SUBIECTUL III: Fie ∆ , mijlocul segmentului [ ], mijlocul segmentului [ ], mijlocul
segmentului [ ]. a. Să se arate că ⃗ = ⃗ + ⃗ . b. Să se arate că ⃗ + ⃗ + ⃗ = ⃗.

Prof. Doru Preșneanu


SUBIECTUL IV: Fie un pătrat și punctul de intersecție al diagonalelor. Să se verifice
relațiile: a. ⃗ + ⃗ + ⃗ + ⃗ = ⃗. b. ⃗= − ⃗+ ⃗+ ⃗ . Prof. Doru Preșneanu

Clasa a X-a PROFIL UMAN


SUBIECTUL I: Fie pătratul cu vârful ( , ) și ecuația laturii : + = .

a.Aflați coordonatele vârfurilor , și în cazul când are ambele coordonate negative.

b.Arătați că diagonalele pătratului se intersectează în originea axelor de coordonate.


c.Aflați coordonatele vectorului ⃗ = ⃗ − ⃗. Prof. Doru Preșneanu
SUBIECTUL II: Fie numerele = √ ∙√ ∙√ ∙…∙√ și

= ∙ ∙ ∙…∙ . a.Aflați ∈ ℕ pentru care = . b.Aflați ∈ ℕ∗


pentru care = Prof. Doru Preșneanu

SUBIECTUL III: Aflați numerele reale și care verifică simultan condițiile:

a. = b. = ∙ . Prof. Doru Preșneanu


SUBIECTUL IV: Fie ecuația − ∙ + + = , ∈ℝ

a.Rezolvați ecuația pentru = .


b.Aflați valorile lui pentru care ecuația are soluții egale. Prof. Doru Preșneanu
Clasa a XI-a PROFIL UMAN
SUBIECTUL I: a.Prețul unui produs se scumpește cu %iar apoi, se ieftinește cu %. Aflați
prețul inițial al produsului.

b.Dintre cei de elevi ai unei clase, elevi participă la cercul de matematică, iar elevi
participă la cercul de limba și literatura română. Știind că fiecare elev participă la cel puțin unul
dintre cele două cercuri, să se determine numărul elevilor care participă atât la cercul de limba și
literatura română cât și la cercul de matematică. Prof. Ovidiu Ticuși
SUBIECTUL II: Fie funcția : ℝ → ℝ având proprietatea că ( ) = − , ∀ ∈ ℝ.

a.Să se verifice dacă ( ) = .


b.Să se verifice dacă funcția este o funcție bijectivă. Prof. Ovidiu Ticuși
SUBIECTUL III: Fie = ( , ∞) și legea de compoziție ∗: × → , ∗ = − − + .

a.Să se arate că legea de compoziție este asociativă, comutativă și cu element neutru.


b.Să se arate că ∗ = ( − )( − ) + , ∀ , ∈ .
c.Să se arate (− ) ∗ (− )∗ ∗ < 2. Prof. Ovidiu Ticuși
( )
SUBIECTUL IV: Fie graficul orientat = , , având mulțimea vârfurilor
= { , , , , , } și mulțimea arcelor
= {( , ), ( , ), ( , ), ( , ), ( , ), ( , ), ( , ), ( , ), ( , ), ( , )}.

a.Să se reprezinte graful sub formă sagitală.

70 REVISTA DE MATEMATICĂ MEHEDINȚEANĂ NR. 15


S.S.M.ROMÂNIA - Filiala Mehedinți 2015
b.Să se determine cinci drumuri având extremitatea inițială vârful și extremitatea finală vârful
. c.Să se determine un circuit hamiltonian. Prof. Ovidiu Ticuși

Clasa a XII-a PROFIL UMAN


SUBIECTUL I: Se consideră matricea ( ) = , ( )∈ ( )

a.Arătațică ( ) ∙ ( ) = ( + ), ∀ , ∈ ℝ. b.Calculați ( ) − ( ).

c.Arătațică ( )− ( ) = . Prof.Gabriela Bondoc

SUBIECTUL II: a.Demonstrați că − ( )∙ +( )∙ = , ∀ ∈ ( ).

b.Dacă = , calculați . c.Folosindpunctul a. rezolvați ecuația = .


− − − −
Prof.Gabriela Bondoc
− −
SUBIECTUL III: a.Fie = , ∈ ℤ, = . Arătați că inversa oricărei matrice
+

este tot o matrice din . b.Folosind proprietățile determinanților calculați .


+ + +
Prof.Gabriela Bondoc

SUBIECTUL IV: Fie punctele (− , ), ( , − ) și ( , + ), ∈ ℤ. a.Demonstrați că


punctele , , nu sunt coliniare pentru nici o valoare a lui . b.Determinați valoare a lui pentru
care aria triunghiului este 2. c.Pentru = − determinați ecuația medianei din .

Prof.Gabriela Bondoc

OLIMPIADA DE MATEMATICĂ-ETAPA LOCALĂ-14.02.2015


Clasa a-V-a
SUBIECTUL I: a. Să se rezolve ecuațiile: ( + + + ⋯+ ): =

b. Să se determine ∈ ℕ știind că numărul divizorilor naturali ai lui

∙ este .

SUBIECTUL II : a. Să se scrie numărul ca sumă a șapte numere naturale consecutive.

b.Să se determine , , încât: + + =

SUBIECTUL III: Arătați că nu există pătrate perfecte de forma cu ≠ .

SUBIECTUL IV : Determinați numerele naturale care prin tăierea ultimei cifre se micșorează de
exact 11 ori.

Clasa a VI-a
SUBIECTUL I: Să se determine cifrele , astfel încât:

,( )+ ,( ) + ⋯+ ,( )= .

71 REVISTA DE MATEMATICĂ MEHEDINȚEANĂ NR. 15


S.S.M.ROMÂNIA - Filiala Mehedinți 2015

SUBIECTUL II:

a. Să se rezolve ecuația : + = + , știind că: ∙ = .


b. Să se rezolve ecuația: + + = + + + ( + + ).

SUBIECTUL III: a.Astăzi este ziua de naștere a unui copil, dar și a bunicului său. Bunicul are atâția
ani câte luni are nepotul său, iar suma vârstelor lor este de 78 ani. Câți ani are fiecare?

b.Fie numărul natural = . Să se arate că dacă = atunci se divide cu .


° ʹ ʹʹ
SUBIECTUL IV : Fie ( bisectoarea unghiului ∢ cu măsura de .

( este bisectoarea ∢ ,( este bisectoarea ∢ ....( este bisectoarea


ʹʹ
∢ . Aflați cel mai mic pentru care (∢ )< .

Clasa a VII-a
SUBIECTUL I: Fie , ,…, puncte pe o dreaptă având proprietatea că distanța dintre
oricare două este strict mai mică decât 1. Să se arate că suma tuturor distanțelor dintre oricare
două puncte este strict mai mică decât 1007 1008.

SUBIECTUL II : Fie ʹ mediană în triunghiul ; ʹ ∈ ( ). Fie ∈ ʹ


; ∈ ʹ . Paralela
prin la ʹ intersectează şi în , respectiv . Paralela prin la ʹ intersectează şi
ʹ
în ʹ, respectiv ʹ. Să se arate că: + = + ʹ.

SUBIECTUL III : Să se rezolve ecuația:


− − − −
+ + + ⋯+ =

SUBIECTUL IV : Fie triunghiul echilateral cu = cm. Fie punctele , , astfel ca

∈( ), ∈( ) și ∩ = { }. Calculați , știind că = = cm.

Clasa a VIII-a
SUBIECTUL I Determinați soluțiile naturale ale ecuației:

√ + + − + √ − = + + +

SUBIECTUL II : a. Să se găsească cel mai mare ∈ ℕ∗ astfel încât:

+ + ⋯+ < 2013√
+√ +√ +√ −
b. Să se demonstreze că:

+ + ⋯+ <1−
√ + √ √ + √ √ + √ √
SUBIECTUL III: Fie o piramidă triunghiulară regulată. Se ridică într-un punct al bazei o
perpendiculară pe planul bazei care intersectează planele fețelor laterale în , , . Să se
demonstreze că suma + + este constantă.

72 REVISTA DE MATEMATICĂ MEHEDINȚEANĂ NR. 15


S.S.M.ROMÂNIA - Filiala Mehedinți 2015
ʹ
SUBIECTUL IV : O prismă hexagonală regulată are baza de arie și aria unei secțiuni diagonale .
Calculați în funcție de , ʹ volumul prismei în toate cazurile posibile.

Clasa a IX-a
SUBIECTUL I : Fie , , , ∈ ( , ∞). Să se arate că:

+ + + + + + + ≥

Prof.Daniel Sitaru-RMM-14/2014
SUBIECTUL II : Se consideră șirul crescător ( ) definit prin:

= , = , = ∙ − + , ≥ . Să se arate că:

= + ∙

GM-10/2014
SUBIECTUL III:a. Dați un exemplu de progresie geometrică neconstantă care are o infinitate de
termeni iraționali. (Justificare)

b. O progresie aritmetică are doi termeni raționali. Demonstrați că toți termenii săi sunt raționali.

c. O progresie aritmetica are doi termeni iraționali. Să se arate că progresia are o infinitate de
termeni iraționali. Prof. Dr.Gheorghe Căiniceanu

SUBIECTUL IV: Se consideră în plan 2015 vectori , ,…, . Doi elevi se joacă
alegând alternativ câte un vector, până când sunt aleși toți vectorii. Pierde jocul cel pentru care
suma vectorilor aleși are lungimea mai mică. Arătați că primul elev poate folosi o strategie prin
care să nu piardă jocul. Prof.Dr.Daniel Stretcu

Clasa a X-a
SUBIECTUL I: Să se arate că dacă ∈ ( , ) ∪ ( , ∞) atunci:

+ + ⋯+ =
∙ ∙ ∙

Prof.Daniel Sitaru-RMM-14/2014

SUBIECTUL II: Să se arate că ecuația + + =( + + ) are:

a. Cel puțin două soluții în ℕ∗ ℕ∗ ℕ∗ cu ≤ ≤ ;

b. O infinitate de soluții în ℤ∗ ℤ∗ ℤ∗ . GM-11/2014

SUBIECTUL III: Să se rezolve în mulțimea numerelor reale sistemul:

+ + =
Prof. Dana Paponiu
+ + =
SUBIECTUL IV: Fie , , ∈ ℂ și ∈ ℂ∗ astfel încât:

73 REVISTA DE MATEMATICĂ MEHEDINȚEANĂ NR. 15


S.S.M.ROMÂNIA - Filiala Mehedinți 2015
| − |=| − − |, | − | = | − − | și | − | = | − − |.

Să se determine valoarea raportului . Prof. Dan Nedeianu

Clasa a XI-a

SUBIECTUL I : Să se determine ∈ (ℝ) astfel încât = .

Prof. Ovidiu Ticuși
SUBIECTUL II : Fie ∈ (ℝ) și matricea transpusă. Știind că + = și

+ = să se calculeze .

GM-11-2014
SUBIECTUL III: Să se calculeze:

√ +
→∞ + +

Prof.Daniel Sitaru -RMM-14-2014


SUBIECTUL IV : Să se calculeze limita șirului:

= −
− √ − + √ −

Prof.Claudia Nănuți-RMM-14-2014

Clasa a XII-a
SUBIECTUL I : Pe mulțimea = (− , ) definim legea de compoziție: ∗ = .

a. Să se arate că ( ,∗) este grup abelian.

b. Să se rezolve ecuația: ∗ ∗ ……∗ = . Prof.Leonard Giugiuc


" "

SUBIECTUL II : Fie : ℝ → ℝ, ( )= .

a. Să se arate că este convexă pe [ , ]

b. Să se demonstreze că ( ) ≤ ( − ) + , ∀ ∈ [ , ]

c. Arătați că:

+ +
( ) <

Prof.Dr.Gheorghe Căiniceanu

74 REVISTA DE MATEMATICĂ MEHEDINȚEANĂ NR. 15


S.S.M.ROMÂNIA - Filiala Mehedinți 2015
SUBIECTUL III: Este funcția :

: ℝ ⟶ ℝ, ( ) = [ ]

injectivă? Prof.Daniel Sitaru


SUBIECTUL IV : Să se calculeze (ℤ , +).

Prof.Daniel Sitaru

REZULTATE
OLIMPIADA DE MATEMATICĂ-ETAPA JUDEȚEANĂ-14.03.2015
Nr. Numele și prenumele Școala Clasa Premiul Profesor
1 Mihăescu Răzvan Andrei C.N.”Țițeica” V I Stretcu Daniel
2 Cioroianu Pavel Rareș C.N.”Țițeica” V II Stretcu Daniel
3 Perpelea Iulia Lic.”Șt.Paulian” V III Pătruțescu Sorin
4 Puiu Gabriela Mara C.N.”Traian” V M Prajea Manuela
5 Zăuleț Vlad C.N.”Odobleja” V M Ticuși Ovidiu
6 Nuică Mihai Călin C.N.”Traian” V M Prajea Manuela
7 Diaconescu Christian Șc.Gimnazială 6 V M Bojneagu Dina
8 Ferne Alexandru C.N.”Traian” V M Prajea Manuela
9 Popescu Narcis Șerban C.N.”Țițeica” V M Stretcu Daniel
10 Sărăcin Iulia Maria Șc.”A.Voinescu” V M Coadă Carmen
11 Bătineanu Robert C.N.”Țițeica” V M Stretcu Daniel
12 Chircu Erika Șc.Gimnazială 11 V M Moclea Adriana
13 Harcău Radu C.N.”Traian” V M Prajea Manuela
14 Mergea Erika Maria C.N.”Traian” V M Prajea Manuela
15 Puiu Mihaela Maria C.N.”Traian” V M Prajea Manuela
16 Achim Maria Șc.Gimnazială 3 V M Ghilerdea Elena
17 Popițu George Alexandru Șc.”P.Dumitriu” V M Vasilcan Tiberiu
18 Băzăvan Sorina Maria C.N.”Țițeica” V M Stretcu Daniel
19 Calu Andrei Daniel C.N.”Țițeica” V M Stretcu Daniel
20 Fota Darius C.N.”Traian” V M Prajea Manuela
21 Screciu Mara C.N.”Traian” V M Prajea Manuela
22 Stoican Radu C.N.”Traian” V M Grecu Vasile

1 Semen Valentin C.N.”Traian” VI I Prajea Manuela


2 Vasile Marian Daniel C.N.”Traian” VI I Prajea Manuela
3 Șoșea Miruna C.N.”Traian” VI II Prajea Manuela
4 Dîrpeș Darius C.N.”Odobleja” VI III Bondoc Gabriela
5 Lunguleasa Eugen C.N.”Traian” VI M Prajea Manuela
6 Alecsandroniu Dragoș C.N.”Odobleja” VI M Bondoc Gabriela
7 Popa Laura C.N.”Odobleja” VI M Bondoc Gabriela
8 Raicu Iulia C.N.”Traian” VI M Prajea Manuela
9 Dorobanțu Andrei C.N.”Traian” VI M Prajea Manuela
10 Mihart Cosmin C.N.”Odobleja” VI M Bondoc Gabriela
11 Apetrei Anastasia C.N.”Traian” VI M Prajea Manuela

75 REVISTA DE MATEMATICĂ MEHEDINȚEANĂ NR. 15


S.S.M.ROMÂNIA - Filiala Mehedinți 2015
12 Ghijan Alexandru C.N.”Odobleja” VI M Bondoc Gabriela
13 Ispas Andrei C.N.”Traian” VI M Prajea Manuela
14 Pârvulescu Anisia C.N.”Traian” VI M Prajea Manuela
15 Tătucu Miruna C.N.”Traian” VI M Prajea Manuela
16 Ardei Delia C.N.”Țițeica” VI M Ungureanu Octavian
17 Drăgan Pavel C.N.”Traian” VI M Prajea Manuela
18 Dumitru Alexandra C.N.”Traian” VI M Prajea Manuela
19 Olaru Denisa C.N.”Traian” VI M Prajea Manuela

1 Crăciunescu Ion Emanuel Șc.”P.Dumitriu” VII I Vasilcanu Tiberiu


2 Bușe Iasmina C.N.”Traian” VII II Căiniceanu George
3 Grecu Bogdan C.N.”Traian” VII III Căiniceanu George
4 Pîlșan Bianca Șc.”A.Voinescu” VII M Coadă Carmen
5 Jianu Diana Florentina C.N.”Țițeica” VII M Draga Tătucu M.
6 Guran Sarah C.N.”Traian” VII M Căiniceanu George
7 Pirvuceanu Alexandru Șc.”A.Voinescu” VII M Coadă Carmen
8 Tîrîși Claudiu Șc.Gimnazială 14 VII M Ionică Constantin

1 Florea Andrei Bogdan C.N.”Țițeica” VIII I Stretcu Daniel


2 Șeitan Radu C.N.”Traian” VIII II Paponiu Dana
3 Picior Cătălin C.N.”Traian” VIII III Paponiu Dana
4 Vladu Cristina Șc.”A.Voinescu” VIII M Coadă Carmen
5 Croitoru Robert C.N.”Traian” VIII M Paponiu Dana
6 Drăcea Iulian Șc.Gimnazială 14 VIII M Palașcă Vladimira

1 Marghescu Bogdan C.N.”Traian” IX I Prajea Manuela


2 Burdescu Alexandru C.N.”Traian” IX II Prajea Manuela
3 Bobîrsc Laura C.N.”Traian” IX III Prajea Manuela
4 Ilioaia Andreea C.N.”Traian” IX M Prajea Manuela
5 Vîjaică Paul C.N.”Traian” IX M Prajea Manuela
6 Morariu Vlad Lic.”T.Lalescu” IX M Farago Alexandru
7 Iacobescu Floarea C.N.”Țițeica” IX M Vasilcanu Florentina
8 Nețoiu Andrei C.N.”Traian” IX M Prajea Manuela
9 Sbîrcea Anidora C.N.”Țițeica” IX M Vasilcanu Florentina
10 Antonescu Denis C.N.”Traian” IX M Prajea Manuela
11 Iordăchescu Anca C.N.”Traian” IX M Prajea Manuela
12 Andrei Eliza C.N.”Traian” IX M Prajea Manuela
13 Drăghici Valentin Gabriel C.N.”Țițeica” IX M Vasilcanu Florentina
14 Mucioniu Ionuț Andrei C.N.”Țițeica” IX M Vasilcanu Florentina
15 Sbîrcea Vlad Robert C.N.”Țițeica” IX M Vasilcanu Florentina

1 Lungu Vlad C.N.”Traian” X I Căiniceanu George


2 Chivără Diana C.N.”Traian” X II Căiniceanu George
3 Găman Eduard C.N.”Traian” X III Căiniceanu George
4 Radu David Alexandru C.N.”Traian” X M Căiniceanu George
5 Bălă Andreea Marina C.N.”Țițeica” X M Vasilcanu Florentina
6 Fleancu Ingrid C.N.”Traian” X M Căiniceanu George
7 Pîrvulescu Cristian C.N.”Odobleja” X M Ticuși Ovidiu

76 REVISTA DE MATEMATICĂ MEHEDINȚEANĂ NR. 15


S.S.M.ROMÂNIA - Filiala Mehedinți 2015
8 Zanfir Andra C.N.”Traian” X M Căiniceanu George
9 Calagiu Andreea C.N.”Țițeica” X M Vasilcanu Florentina
10 Priboi Mălina C.N.”Traian” X M Căiniceanu George
11 Șerban Bianca C.N.”Țițeica” X M Vasilcanu Florentina
12 Spîrlea Ionuț C.N.”Odobleja” X M Ticuși Ovidiu

1 Meterez Alexandru C.N.”Țițeica” XI I Draga Tătucu M.


2 Popescu Cristiana C.N.”Traian” XI II Căiniceanu George
3 Badea Simona C.N.”Țițeica” XI III Draga Tătucu M.
4 Oichea Adrian C.N.”Țițeica” XI III Draga Tătucu M.
5 Firuți Bogdan C.N.”Traian” XI M Căiniceanu George
6 Opranescu Veronica C.N.”Țițeica” XI M Draga Tătucu M.
7 Protopopescu Ruri C.N.”Traian” XI M Vasile Tomiță
8 Sandu Gabriel C.N.”Țițeica” XI M Stretcu Daniel
9 Tipsie Andrei Gabriel C.N.”Țițeica” XI M Nedeianu Dan

1 Neciu Denisa C.N.”Țițeica” XII I Draga Tătucu M.


2 Glăvan Steliana C.N.”Țițeica” XII II Draga Tătucu M.
3 Nicolicioiu Armand C.N.”Țițeica” XII II Draga Tătucu M.
4 Bărzuică Alexandru C.N.”Țițeica” XII M Draga Tătucu M.
5 Cioancă Ramona Mihaela C.N.”Odobleja” XII M Bălu Nicoleta
6 Enculescu Răzvan C.N.”Țițeica” XII M Draga Tătucu M.
CONCURSUL NAȚIONAL DE MATEMATICĂ APLICATĂ „ADOLF HAIMOVICI”
ETAPA JUDEȚEANĂ-14.03.2015
Nr. Numele și prenumele Școala Clasa Premiul Profesor
1 Marinescu Adrian C.N.”Odobleja” IX I Bălu Nicoleta
2 Papavă Patricia Daniela Lic.”T.Lalescu” IX II Tudose Elena
3 Voicu Livia Inela C.N.”Odobleja” IX III Bălu Nicoleta
4 Negrescu Maria Ionela C.N.”Odobleja” IX M Bălu Nicoleta
5 Oana Elena Andra C.N.”Odobleja” IX M Bălu Nicoleta
6 Giurescu Mălina C.N.”Traian” IX I Gimoiu Iuliana
7 Neo Ionuț Gabriel C.N.”Țițeica” IX III Vasilcanu Florentina
8 Dăogaru Claudiu Catalin C.N.”Țițeica” IX M Vasilcanu Florentina
9 Degetaru Narcis Florin C.N.”T.Costescu” IX II Lădaru Daniela
10 Rolea Roxana C.N.”T.Costescu” IX III Lădaru Daniela
11 Ciortan Bogdan C.N.”T.Costescu” IX M Sitaru Daniel
12 Neamțu Ioana Andreea C.N.”T.Costescu” IX M Lădaru Daniela
13 Fluerașu Cosmin C.N.”T.Costescu” IX M Sitaru Daniel
14 Brebu Costinel Gabriel C.N.”T.Costescu” IX M Sitaru Daniel
15 Tătucu Alin C.N.”T.Costescu” IX M Ladaru Daniela
16 Costăchioiu Andreea C.N.”Traian” X III Giugiuc Leonard
17 Dron Robert Sebastian C.T.”Dierna” X I Vuc Ionelia
18 Cliseru Georgiana C.N.”T.Costescu” X II Sitaru Daniel
19 Rostogol Maria Denisa C.N.”Traian” X I Giugiuc Leonard
20 Gomoi Viviana C.N.”Odobleja” X III Ticuși Ovidiu
21 Moți Iasmina Andreea C.N.”Traian” X III Giugiuc Leonard
22 Seimeanu Andrei C.N.”Odobleja” X M Ticuși Ovidiu

77 REVISTA DE MATEMATICĂ MEHEDINȚEANĂ NR. 15


S.S.M.ROMÂNIA - Filiala Mehedinți 2015
23 Dumitrașcu Ana Maria Lic.”T.Lalescu” XI III Gorun Sanda
24 Tobă Alexandra C.N.”Traian” XI M Pupăză Ecaterina
25 Lungu Georgiana C.N.”T.Costescu” XII III Sitaru Daniel
26 Gheran Daniel Vlăduț C.N.”T.Costescu” XII M Lădaru Daniela
27 Al-Eni Petronela C.N.”T.Costescu” XII M Sitaru Daniel
28 Butaru Daniel Florian C.N.”Odobleja” XII I Bălu Nicoleta
29 Ungureanu Dragoș C.N.”Odobleja” XII II Bălu Nicoleta
30 Voicu Iuliana C.N.”Odobleja” XII III Ticuși Ovidiu
31 Dorobanțu Adina C.T.”Dierna” XII I Fallon Florica
32 Bușoi Ileana Gabriela C.N.”Țițeica” XII I Nedeianu Dan
33 Duțu Elena Camelia C.N.”Odobleja” XII II Bondoc Gabriela
34 Lunguleasa Ionelia C.N.”Odobleja” XII III Bondoc Gabriela
35 Papa Ramona Dariana C.N.”Traian” XII M Pupăză Ecaterina
36 Grecu Irina Maria C.N.”Odobleja” XII I Bondoc Gabriela
37 Mladenovici Ionela C.N.”Odobleja” XII II Bălu Nicoleta
38 Al-Badi Maria Cosmina C.N.”Odobleja” XII III Bălu Nicoleta
39 Blendea Irina C.N.”Odobleja” XII M Bălu Nicoleta
OLIMPIADA NAȚIONALĂ DE MATEMATICĂ-2015
1 Crăciunescu Ion Emanuel Șc.”P.Dumitriu” VII MENTIUNE Vasilcanu Tiberiu
MECS
AUR-SSMR
2 Marghescu Bogdan C.N.”Traian” IX BRONZ- Prajea Manuela
SSMR
3 Neciu Denisa C.N.”Țițeica” XII BRONZ- Draga Tătucu M.
SSMR
CONCURSUL NAȚIONAL DE MATEMATICĂ APLICATĂ „ADOLF HAIMOVICI”
ETAPA NAȚIONALĂ-2015
1 Dron Robert Sebastian C.T.”Dierna” X II Vuc Ionelia
2 Grecu Irina Maria C.N.”Odobleja” XII III Bondoc Gabriela
3 Giurescu Mălina C.N.”Traian” IX M Gimoiu Iuliana
4 Dorobanțu Adina C.T.”Dierna” XII M Fallon Florica
5 Papavă Patricia Daniela Lic.”T.Lalescu” IX M Tudose Elena
6 Mladenovici Ionela C.N.”Odobleja” XII M Bălu Nicoleta
REZULTATE PURPLE COMET-MATH MEET-2015
1 Cenușe Andrada C.N.”Traian” VIII MENTIUNE Paponiu Dana
2 Dumitrache Francesca Echipaj DST1
3 Fulga Fabian Middle School
4 Picior Cătălin
5 Șeitan Radu
6 Ștefan Bogdan
7 Iordăchescu Vlad Șc.Gimnazială nr.5 VIII MENTIUNE Piț-Rada Marica
8 Paulescu Andrei Echipaj PENTAGON
9 Neagoe Ramona Middle School
10 Stoianovici Bianca
11 Savu Teodora

78 REVISTA DE MATEMATICĂ MEHEDINȚEANĂ NR. 15


S.S.M.ROMÂNIA - Filiala Mehedinți 2015

AUTORII MATERIALELOR DIN R.M.M.-15


Nr.crt. Numele și prenumele Nr.crt. Numele și prenumele
1 DANIEL SITARU 16 IULIANA GIMOIU
2 GHEORGHE CĂINICEANU 17 DORIN MĂRGHIDANU
3 ION NĂNUȚI 18 DOREL CĂPRARU
4 MIHAI OCTAVIAN UNGUREANU 19 BĂLĂȘOIU DANIELA
5 DAN NĂNUȚI 20 OVIDIU TICUȘI
6 LEONARD GIUGIUC 21 CONSTANTIN IONICĂ
7 EMILIA RĂDUCAN 22 GHEORGHITA TUFIȘ
8 CLAUDIA NĂNUȚI 23 ELENA RÎMNICEANU
9 DIANA TRĂILESCU 24 DANA PAPONIU
10 MARIA UNGUREANU 25 DRAGA TĂTUCU MARIANA
11 ELENA GHILERDEA 26 PIȚ RADA MARICA
12 ECATERINA PUPĂZĂ 27 PIȚ RADA IONEL VASILE
13 MARIANA CORNEA 28 DAN NEDEIANU
14 GABRIELA BONDOC 29 DANIEL STRETCU
15 ALEXANDRU TOPORAN 30 MARICA ȘTEFAN

NOTĂ: Website-ul SSMR-Filiala Mehedinți are adresa http//:www.ssmrmh.ro


Pe acest site găsiți conținutul numerelor anterioare ale RMM, subiectele de la OLM,
OJM,ONM,baraje,OBM,OIM, Concursul Național de Matematică Aplicată”Adolf
Haimovici”, baza de date cu probleme rezolvate, note și articole matematice, rezolvările
problemelor din RMM precedente, clasamentul rezolvitorilor GMB și galeria de onoare
cuprinzând elevii medaliați la olimpiadele naționale și internaționale pe care suntem
mândri ca i-am cunoscut. Pentru a publica probleme propuse, articole și note matematice
în RMM sau pe site puteți trimite materialele pe mailul: dansitaru63@yahoo.com

79 REVISTA DE MATEMATICĂ MEHEDINȚEANĂ NR. 15

S-ar putea să vă placă și